Download LeVVon 11.1 Fundamental Theorem of Counting

Survey
yes no Was this document useful for you?
   Thank you for your participation!

* Your assessment is very important for improving the workof artificial intelligence, which forms the content of this project

Document related concepts
no text concepts found
Transcript
LESSON OBJECTIVES
&KDSWHU
11.1 Fundamental Theorem of
Counting
• Use the Fundamental
Theorem of Counting to
find probabilities.
11.2 Compound Events
• Find the probability of
compound events.
11.3 Collecting Data
• Analyze methods of
gathering information.
• Design an appropriate
study to gather
information.
11.4 Probability Models
• Use probability to model
random events.
• Use expected value to
make decisions.
&RQWHQWV
11.5 The Normal Distribution
• Use the Empirical Rule to
find probabilities.
• Find z-scores and use
the standard normal
distribution.
)XQGDPHQWDO7KHRUHP
RI&RXQWLQJ
&RPSRXQG(YHQWV
&ROOHFWLQJ'DWD
3UREDELOLW\'DWD
7KH1RUPDO'LVWULEXWLRQ
0DWK/DEV
6SUHDGRIDQ,QIHFWLRQ
5RFN3DSHU6FLVVRUV
0DWK$SSOLFDWLRQV
&KDSWHU5HYLHZ
&KDSWHU$VVHVVPHQW
&KDSWHU3UREDELOLW\DQG6WDWLVWLFV
Mental Math
Skills Review
Multiply.
Simplify each expression.
1. 5 + 11 8
18 18 9
2. 1 + 1 − 1 7
4 2 20 10
3
3. • 8 6
4 25 25
7
1
45
4.
28 4
45
1.4 • 5 • 2 40
2.2 • 3 • 6 36
3.4 • 7 • 2 • 3 168
4.5 • 8 • 3 • 9 1,080
506 Chapter 11 Probability and Statistics
3UREDELOLW\DQG6WDWLVWLFV
:K\VKRXOG,OHDUQWKLV"
:
3URE
3UREDELOLW\LVDUDWLRWKDWTXDQWL¿HVWKHOLNHOLKRRGRIDSDUWLFXODUHYHQW
,QWHUSUHWLQJSUREDELOLWLHVKHOSVWRSUHSDUHLQGLYLGXDOVIRUGDLO\OLIHHYHQWVVXFKDV
,QWHU
WKHZHDWKHULQYHVWPHQWRSSRUWXQLWLHVDQGFRPSHWLWLRQV
WKH
$QDO\]LQJVWDWLVWLFVDQGSUREDELOLWLHVDQGPDNLQJLQIRUPHGGHFLVLRQVDUHDPRQJ
$Q
WKHUHVSRQVLELOLWLHVRISURIHVVLRQDOVWKDWKDQGOHGDWD
WKH
‡ &ULPH6FHQH,QYHVWLJDWRUVSURFHVVGDWDJDWKHUHGWRFRPSXWH
SUREDELOLWLHVWKDWFDQOHDGWRVROYLQJFULPHV
‡ 0DUNHWHUVDQG5HWDLOHUVXVHVWDWLVWLFVWRGHVLJQVWRUHGLVSOD\V
‡ &KHPLVWVXVHSUREDELOLWLHVWRFRPSDUHGLIIHUHQWFRPSRXQGV
,QWKLVFKDSWHU\RXZLOOFRPSXWHWKHSUREDELOLW\RIFRPSRXQGHYHQWVDQG
XVHVWDWLVWLFVWRVROYHSUREOHPVLQYROYLQJSUREDELOLW\
3URMHFW,GHD0DNHD)DVKLRQ6WDWHPHQW
0RVWFORWKLQJLWHPVLQ\RXUFORVHWFDQEHZRUQDVSDUWRIPRUHWKDQ
RQHRXWILW)DVKLRQFRQVXOWDQWVUHFRPPHQGWKDWHYHU\RQHKDYHDIHZ
FRUHLWHPVWKDWFDQEHZRUQDVSDUWRIPDQ\RXWILWV7KHPRUHEDVLF
FRORUV\RXKDYHLQ\RXUZDUGUREHWKHPRUHYHUVDWLOH\RXUZDUGUREH
%XLOGDVDPSOHYHUVDWLOHZDUGUREHXVLQJFORWKHV\RXDOUHDG\RZQDQG
DIHZQHZRQHV6HOHFWILYHDUWLFOHVRIFORWKLQJIURP\RXUFORVHW<RX
FDQMXVWOLVWWKHFORWKHVZLWKGHVFULSWLRQV)RUWKHQHZFORWKHVXVH
FDWDORJVDQGVDOHIO\HUVWRLGHQWLI\WKUHHDUWLFOHVRIFORWKLQJWKDWFRXOG
EHSXUFKDVHGIRUDWRWDORIRUOHVVWRDGGWR\RXUZDUGUREH8VH
FRPELQDWLRQVWRGHWHUPLQHWKHQXPEHURIGLIIHUHQWRXWILWV\RXUHLJKW
DUWLFOHVRIFORWKLQJFDQPDNH3ODQ\RXUGDLO\DWWLUHIRUDVPDQ\GD\V
DV\RXFDQEHIRUH\RXKDYHWRUHSHDWDQH[DFWRXWILW
&KDSWHU3UREDELOLW\DQG6WDWLVWLFV
Look to Your Future
Actuaries collect and scrutinize data to approximate the probability of an
event. Actuaries work for the insurance industry where probabilities of events,
such as illness, disability, or loss of property are calculated as a risk. The
probability or risk of such an event determines what policies the company can
issue and at what cost in order to remain profitable. PLANNING
THE CHAPTER
Math Labs, pp. 538–541
Data Sheet (Lab Data Sheets)
Math Applications, pp. 542–549
Chapter Review, pp. 550–551
Chapter Test, p. 552
Software Generated Assessment
Standardized Test Practice, p. 553
Grid Response Form (CRB)
Chapter Resource Book (CRB)
Reteaching, pp. 393, 397,
403, 407
Extra Practice, pp. 395, 399,
405, 409
Enrichment, pp. 401, 411
Standardized Test Response
Form, pp. 413, 414
Standardized Test Answers,
p. 415
Classroom/Journal
Topics
What’s Ahead?
In this chapter, students will
learn how to calculate the
probability of compound events.
The Fundamental Theorem of
Counting is used in many real-life
situations to find probabilities.
Students should become familiar
with probability models and the
normal distribution.
Chapter 11 Probability and Statistics
507
LESSON PLANNING
/HVVRQ )XQGDPHQWDO7KHRUHP
/
RI&RXQWLQJ
2EMHFWLYHV
2
Vocabulary
Fundamental Theorem of
Counting
independent events
dependent events
8VHWKH)XQGDPHQWDO
7KHRUHPRI&RXQWLQJWR
ILQGSUREDELOLWLHV
Extra Resources
Reteaching 11.1
Extra Practice 11.1
$ODQLVSOD\LQJDFDUGJDPHZLWK
DIULHQG+HSLFNVRQHFDUGIURP
DGHFNRIFDUGVDQGWKHQSLFNV
DQRWKHUFDUGZLWKRXWUHSODFLQJWKH
¿UVW$ODQQHHGVWRSLFNWZRKHDUWV
LQDURZWRZLQWKHJDPH:KDWLV
WKHSUREDELOLW\WKDW$ODQZLOOZLQ
WKHJDPH"
)XQGDPHQWDO7KHRUHPRI&RXQWLQJ
Assignment
)XQGDPHQWDO7KHRUHPRI&RXQWLQJ
,IRQHHYHQWFDQRFFXULQPGLIIHUHQWZD\VDQGDQRWKHU
HYHQWFDQRFFXULQQGLIIHUHQWZD\VWKHQWKHUHDUH
P‡QGLIIHUHQWZD\VIRUERWKHYHQWVWRRFFXU
In-class practice: 1–5
Homework: 6–26
Math Applications
7KH)XQGDPHQWDO7KHRUHPRI&RXQWLQJFDQEHXVHGWR¿QGWKH
QXPEHURISRVVLEOHRXWFRPHVRIGLIIHUHQWHYHQWV
Exercises 4, 6, and 9 from
pages 542–549
([DPSOH 8VLQJWKH)XQGDPHQWDO7KHRUHP
RI&RXQWLQJ
START UP
7KHFDIHWHULDDWDQRI¿FHFRPSOH[VHUYHVWKUHHGLIIHUHQWW\SHVRI
VDODGVWKUHHGLIIHUHQWW\SHVRIVRXSVDQGVL[GLIIHUHQWW\SHVRIVDODG
GUHVVLQJV+RZPDQ\GLIIHUHQWRUGHUVXVLQJRQHVRXSRQHW\SHRI
VDODGDQGRQHW\SHRIVDODGGUHVVLQJDUHSRVVLEOH"
Remind students that tree
diagrams and organized lists
are good strategies for problem
solving. Ask students to make
either a tree diagram or an
organized list to represent all of
the possible orders for Example 1.
6ROXWLRQ
8VHWKH)XQGDPHQWDO7KHRUHPRI&RXQWLQJWR¿QGWKHWRWDOQXPEHU
RIRUGHUV
‡‡ 7KHUHDUHGLIIHUHQWRUGHUVRIVRXSDQGVDODGZLWKRQHW\SHRI
VDODGGUHVVLQJ
INSTRUCTION
Use the students’ tree
diagrams and organized lists
to help students develop
the Fundamental Counting
Principal. After the students have
expressed the mechanics of the
Fundamental Counting Principal
in their own words, present the
formal notation.
Example 1 Extend the
problem by modifying it
to include three choices of
beverages. Emphasize to students
that they can use the answer to
the original problem to obtain the
answer to the modified problem.
&KDSWHU3UREDELOLW\DQG6WDWLVWLFV
R.E.A.C.T. Strategy
Cooperating
Instruct students to write a Fundamental Counting Principal problem. Select a
partner for each student. Students should solve their partner’s problem. Then
have students write a second problem that is a modification of the problem
they just solved (their partner’s problem). After the second problems are
written, ask students to exchange papers and solve the problem written by
their partner.
508 Chapter 11 Probability and Statistics
INSTRUCTION
2QJRLQJ$VVHVVPHQW
Ask students to provide examples
of other events which would
be considered independent. In
contrast, ask students to provide
examples of events which would
not be considered independent.
$QXUVHU\VHOOVGLIIHUHQWW\SHV
RIÀRZHUVGLIIHUHQWW\SHV
RIEXVKHVDQGGLIIHUHQWW\SHV
RIWUHHV+RZPDQ\GLIIHUHQW
ZD\VFDQDFXVWRPHURUGHU
ÀRZHUEXVKDQGWUHH"
'HSHQGHQWDQG,QGHSHQGHQW(YHQWV
Tell students that experimental
probability is the surveyed
chance of an event occurring,
while theoretical probability is
the mathematical or calculated
chance of an event occurring.
Point out that there is no way to
calculate a theoretical probability
concerning free throw attempts.
0DUWLQDUROOHGDQXPEHUFXEHDQGÀLSSHGDFRLQ%HFDXVHWKHRXWFRPHVRI
HDFKHYHQWDUHQRWLQÀXHQFHGE\HDFKRWKHUWKH\DUHLQGHSHQGHQWHYHQWV
3UREDELOLW\RI,QGHSHQGHQW(YHQWV
,I$DQG%DUHLQGHSHQGHQWHYHQWVWKHQWKHSUREDELOLW\
WKDWERWK$DQG%ZLOORFFXULVJLYHQE\WKHIRUPXOD
3$DQG% 3$‡3%
([DPSOH 3UREDELOLW\RI,QGHSHQGHQW(YHQWV
0ROO\LVRQWKHKLJKVFKRROEDVNHWEDOOWHDP6KHLVDQIUHHWKURZ
VKRRWHU7KLVPHDQVWKDWWKHH[SHULPHQWDOSUREDELOLW\WKDW0ROO\ZLOO
PDNHDIUHHWKURZLV:KDWLVWKHSUREDELOLW\WKDW0ROO\ZLOO
VXFFHVVIXOO\PDNHDOOWKUHHRIKHUQH[WWKUHHIUHHWKURZV"5RXQG\RXU
DQVZHUWRWKHQHDUHVWWKRXVDQGWK
6ROXWLRQ
(DFKIUHHWKURZDWWHPSWLVDQLQGHSHQGHQWHYHQWEHFDXVHWKHRXWFRPHRI
RQHDWWHPSWGRHVQRWDIIHFWWKHRXWFRPHRIDQRWKHU
3PDNHDOO ‡‡
3PDNHDOO§
7KHSUREDELOLW\WKDW0ROO\ZLOOVXFFHVVIXOO\PDNHHDFKRIKHUQH[WWKUHHIUHH
WKURZVLVDERXWRU
2QJRLQJ$VVHVVPHQW
$WDFDUQLYDOGXFNSRQGJDPHWKHUHDUHUXEEHUGXFNVÀRDWLQJRQ
WKHZDWHU7ZHQW\¿YHRIWKHGXFNVDUHPDUNHG³:LQQHU´RQWKHERWWRP
$SOD\HUUDQGRPO\VHOHFWVDGXFNDQGFKHFNVWRVHHLIKHRUVKHZLQVDSUL]H
7KHQWKHGXFNLVUHWXUQHGWRWKHZDWHU:KDWLVWKHSUREDELOLW\WKDWWKHQH[W
WZRSOD\HUVZLOOZLQDSUL]H"([SUHVVWKHDQVZHUDVDIUDFWLRQ 1
36
)XQGDPHQWDO7KHRUHPRI&RXQWLQJ
Chalkboard Examples
Ask students to name two events A and B, such that:
1. P(A and B) = 1 sample answer: Event A: a card is pulled from a
standard deck and the card is either red or black; Event B: a standard
six-sided die is rolled and a number less than 7 lands face up
2. P(A and B) = 0 sample answer: Event A: a card is pulled from a
standard deck and the card is either red or black; Event B: a standard
six-sided die is rolled and a number greater than 6 lands face up
3. P(A and B) = 0.25 sample answer: Event A: a card is pulled from a
standard deck and the card is red; Event B: a standard six-sided die is
rolled and an even number lands face up
11.1 Fundamental Theorem of Counting 509
INSTRUCTION
:KHQWKHRXWFRPHRIRQHHYHQWDIIHFWVWKHRXWFRPHRIDQRWKHUHYHQWWKH
HYHQWVDUHGHSHQGHQWHYHQWV)RUH[DPSOHWKHUHDUHFDUGVLQDVWDFN
QXPEHUHGIURPWR-XOLRVHOHFWVDFDUGDQGGRHVQRWSXWWKHFDUGEDFNLQ
WKHVWDFN7KHQ*LQDVHOHFWVDFDUG7KHRXWFRPHRI*LQD¶VVHOHFWLRQZLOOEH
LQÀXHQFHGE\WKHRXWFRPHRI-XOLR¶VVHOHFWLRQ
Have students provide examples
of other events which would
be considered dependent.
To reinforce vocabulary
comprehension, students should
justify why the events are not
independent.
3UREDELOLW\RI'HSHQGHQW(YHQWV
,I$DQG%DUHGHSHQGHQWHYHQWVDQG%IROORZV$WKHQ
WKHSUREDELOLW\WKDWERWK$DQG%ZLOORFFXULVJLYHQE\
WKHIRUPXOD
Example 3 An alternate way
3$DQG% 3$‡3%_$
to explain this problem stresses
the use of the Fundamental
Counting Principal. Explain that:
3%_$LVWKHSUREDELOLW\RI%JLYHQWKDW$KDVRFFXUUHG
([DPSOH 3UREDELOLW\RI'HSHQGHQW(YHQWV
P(both spades) = (number of
ways two spades can be chosen
from a deck) divided by (number
of ways two cards can be chosen
from a deck).
•
P(both spades) = 13 12 = 1
52 • 51 17
$WWKHEHJLQQLQJRIWKLVOHVVRQWKHUHZDVDGHVFULSWLRQRIDFDUGJDPH
:KDWLVWKHSUREDELOLW\WKDW$ODQZLOOZLQWKHJDPH"([SUHVVWKHSUREDELOLW\
DVDIUDFWLRQ
6ROXWLRQ
$VWDQGDUGGHFNRISOD\LQJFDUGVKDVKHDUWVVSDGHVGLDPRQGV
DQGFOXEV%HFDXVHWKH¿UVWFDUGLVQRWUHSODFHGWKHRXWFRPHRIWKH
VHFRQGFDUGLVGHSHQGHQWRQWKHRXWFRPHRIWKH¿UVWFDUG
3KHDUWRQ¿UVWSLFN 13 = 1
52 4
$VVXPLQJWKDWDKHDUWLVVHOHFWHGRQWKH¿UVWSLFNWKHUHZLOOEHKHDUWVOHIW
LQWKHUHPDLQLQJFDUGV
Reteaching 11.1 (CRB)

3KHDUWRQVHFRQGSLFNDIWHUKHDUWLVSLFNHG 12
51
1 12
3ERWKKHDUWV 4 51
3ERWKKHDUWV 3 = 1
51 17
7KHSUREDELOLW\WKDW$ODQZLOOZLQWKHJDPHLV 1 17
2QJRLQJ$VVHVVPHQW
7KHUHDUHUHGPDUEOHVEOXHPDUEOHVJUHHQPDUEOHVDQG\HOORZ
PDUEOHVLQDMDU&DUOWRQVHOHFWVDPDUEOHDWUDQGRPDQGGRHVQRWUHSODFHWKH
PDUEOH7KHQKHVHOHFWVDQRWKHUPDUEOH:KDWLVWKHSUREDELOLW\WKDW&DUOWRQ
ZLOOVHOHFWUHGPDUEOHV"([SUHVVWKHDQVZHUDVDIUDFWLRQ 3
95
&KDSWHU3UREDELOLW\DQG6WDWLVWLFV
Diversity in the Classroom
Kinesthetic Learner
Divide students into groups to play the card game in Example 3. Each group
should play the game 100 times and record the results of each game. Each
group should use these results to calculate the experimental probability that
Alan will win the game. Finally, pool the results of each group in the class in
order to calculate the experimental probability that Alan will win the game.
Ask students to explain why the pooled results are closer to the theoretical
probability than the results of each individual group.
510 Chapter 11 Probability and Statistics
Problem Solving
Have students use pencil and
paper or Geometer’s Sketchpad
to draw a scale picture of the
dartboard.
VHHPDUJLQ
$FLUFXODUGDUWERDUGKDVDGLDPHWHURILQFKHV
DQGDFLUFXODUEXOO·VH\HZLWKDGLDPHWHURI
3
4 LQFK,I5HJJLHWKURZVDGDUWDQGLWLVHTXDOO\
OLNHO\WRODQGDQ\ZKHUHRQWKHERDUGZKDWLV
WKHSUREDELOLW\RIKLWWLQJWKHEXOO·VH\H"
6WHS&DUU\2XWWKH3ODQ
8VHWKHIRUPXODIRUWKHDUHDRIDFLUFOH$ ʌU
&RPSDUHWKHDUHDRIWKHEXOO·VH\HWRWKHDUHD
RIWKHFLUFXODUGDUWERDUG([SUHVVWKHSUREDELOLW\
DVDSHUFHQW
6WHS8QGHUVWDQGWKH3UREOHP
'HVFULEHWKHDVSHFWVRIWKHSUREOHPVLWXDWLRQ
:KDWDUH\RXEHLQJDVNHGWRILQG"
6WHS&KHFNWKH5HVXOWV
'RHV\RXUDQVZHUVHHPUHDVRQDEOH"$ERXW
KRZPDQ\WLPHVODUJHUWKDQWKHEXOO·VH\HLVWKH
HQWLUHGDUWERDUG"
6WHS'HYHORSD3ODQ
3UREOHPVROYLQJVWUDWHJ\8VHDIRUPXOD
7KHSUREDELOLW\RIDQHYHQWLVWKHTXRWLHQWRIWKH
QXPEHURISRVVLEOHVXFFHVVHVDQGWKHQXPEHU
RISRVVLEOHRXWFRPHV+RZFDQ\RXXVHDQDUHD
PRGHOWRILQGWKHSUREDELOLW\RIWKHGDUWODQGLQJ
RQWKHEXOO·VH\H"
Ask students if the answer to the
problem would change if the
3 -inch circle were not in the
4
center of the dartboard.
Ask students to calculate the
probability of not hitting the
bull’s-eye of the dartboard.
Emphasize that there are two
different ways of performing this
calculation.
Understand the Problem
You are given the size of a
dartboard and its bull’s-eye. You
want to find the probability that
a randomly thrown dart will land
on the bull’s-eye.
/HVVRQ$VVHVVPHQW
7KLQNDQG'LVFXVVVHHPDUJLQ
([SODLQWKH)XQGDPHQWDO7KHRUHPRI&RXQWLQJ
:KDWGRHVLWPHDQIRUWZRHYHQWVWREHGHSHQGHQW"
:KDWGRHVLWPHDQIRUWZRHYHQWVWREHLQGHSHQGHQW"
+RZFDQ\RXILQGWKHSUREDELOLW\WKDWERWKHYHQWV$DQG%ZLOO
RFFXULIWKHHYHQWVDUHLQGHSHQGHQW"
+RZFDQ\RXILQGWKHSUREDELOLW\WKDWERWKHYHQWV$DQG%ZLOO
RFFXULIWKHRXWFRPHRI%GHSHQGVRQWKHRXWFRPHRI$"
)XQGDPHQWDO7KHRUHPRI&RXQWLQJ
Think and Discuss Answers
Develop a Plan
Divide the area of the bull’s-eye
by the area of the dartboard.
Carry Out the Plan
2
•
P(bull’s-eye) = π 7.5 2
π • 0.375
= 0.0025 = 0.25%
Check the Results
Check: the answer seems
reasonable. The dartboard is a
few hundred times the size of the
bull’s-eye.
1. Answers will vary. Sample answer: if one event can occur m ways and
another event can occur n ways, then both events can occur in m × n
ways.
2. the outcome of one event affects the outcome of the other event
3. the outcomes of the events have no effect on each other
4. multiply the probabilities
5. multiply the probability of A by the probability of B given that A has
already occurred
11.1 Fundamental Theorem of Counting 511
WRAP UP
3UDFWLFHDQG3UREOHP6ROYLQJ
To ensure mastery of objectives,
students should be able to:
• Use the Fundamental Counting
Principal to find the number of
outcomes in each situation.
• Use the Fundamental
Counting Principal to calculate
probabilities.
8VHWKH)XQGDPHQWDO7KHRUHPRI&RXQWLQJWRILQGWKHQXPEHU
RIRXWFRPHVLQHDFKVLWXDWLRQ
Assignment
In-class practice: 1–5
Homework: 6–26
Math Applications
Exercises 4, 6, and 9 from pages
542–549
VKLUWVZKLWHEODFNEURZQEOXH
SDQWVMHDQVVODFNV
+RZPDQ\GLIIHUHQWRXWILWVDUHWKHUHLIRQHFRORUVKLUWDQGRQH
W\SHRISDQWVDUHFKRVHQ"
QXPEHUFXEH
FRLQKHDGVWDLOV
+RZPDQ\GLIIHUHQWRXWFRPHVDUHWKHUHLIDQXPEHUFXEHLVUROOHG
DQGDFRLQLVIOLSSHG"
FUXVWWKLQSDQGHHSGLVK
WRSSLQJVKDPVDXVDJHSHSSHURQLJUHHQSHSSHUVROLYHV
PXVKURRPV
+RZPDQ\GLIIHUHQWSL]]DVDUHWKHUHLI\RXFKRRVHRQHW\SHRI
FUXVWDQGRQHWRSSLQJ"
VRXSWRPDWRFKLFNHQQRRGOHYHJHWDEOH
VDQGZLFKWXUNH\KDPWXQD
GULQNVRIWGULQNWHDZDWHU
+RZPDQ\GLIIHUHQWOXQFKHVDUHWKHUHLIW\SHRIVRXSRQHW\SH
RIVDQGZLFKDQGRQHW\SHRIGULQNDUHFKRVHQ"
8VHWKHWZRVSLQQHUVWRILQGWKHSUREDELOLW\RIHDFKVHW
RIHYHQWV
Extra Practice 11.1 (CRB)

1
3DQG& 40
1
3HYHQQXPEHUDQG$ 10
3
3RGGQXPEHUDQGYRZHO 51 3QXPEHUOHVVWKDQDQG' 20
9
3QXPEHUJUHDWHUWKDQDQGFRQVRQDQW 40
&KDSWHU3UREDELOLW\DQG6WDWLVWLFV
R.E.A.C.T. Strategy
Relating
Ask students to design a 15-inch dartboard such that the probability of hitting
the bull’s-eye would be equal to the probability of not hitting the bull’s-eye.
Encourage students to write an equation as part of the solution process rather
than using guess and check.
512 Chapter 11 Probability and Statistics
Mixed Review
Additional Answers
$EDJFRQWDLQVJUHHQPDUEOHVEOXHPDUEOHV\HOORZ
PDUEOHDQGUHGPDUEOHV0DULWDVHOHFWVRQHPDUEOHDW
UDQGRPDQGGRHVQRWUHSODFHLW7KHQVKHVHOHFWVDVHFRQG
PDUEOHDWUDQGRP)LQGHDFKSUREDELOLW\
1
3JUHHQPDUEOHWKHQD\HOORZPDUEOH 30
2
3JUHHQPDUEOHWKHQDEOXHPDUEOH 15
2
3WZREOXHPDUEOHV 15
4
3EOXHPDUEOHWKHQDUHGPDUEOH 45
3WZR\HOORZPDUEOHV
1
3UHGPDUEOHWKHQDJUHHQPDUEOH 15
$WLUHVWRUHRIIHUVGLIIHUHQWVL]HV
GLIIHUHQWJUDGHVGLIIHUHQWW\SHV
RIFRQVWUXFWLRQDQGGLIIHUHQWVW\OHV
RIWLUHV+RZPDQ\RSWLRQVIRUQHZ
WLUHVGRHVWKHVWRUHRIIHU"
5RQQLHSOD\VEDVHEDOO+HKDV
DQRQEDVHSHUFHQWDJHRI
7KLVPHDQVWKDWWKHH[SHULPHQWDO
SUREDELOLW\WKDWKHZLOOJHWRQEDVH
ZLWKHDFKDWEDWLV:KDWLVWKH
SUREDELOLW\WKDW5RQQLHZLOOJHWRQ
EDVHLQHDFKRIKLVQH[WIRXUDWEDWV"
5RXQGWRWKHQHDUHVWWKRXVDQGWK
26.
Answers to Math
Applications
(DFKTXHVWLRQRQDPXOWLSOHFKRLFHWHVWKDVIRXUDQVZHUFKRLFHV
:KDWLVWKHSUREDELOLW\RIJHWWLQJWKHILUVWTXHVWLRQVFRUUHFWLI
1
\RXJXHVVWKHDQVZHUV"([SUHVVWKHDQVZHUDVDIUDFWLRQ 64
0L[HG5HYLHZ
)LQGDOOWKHUHDOIRXUWKURRWVRIHDFKQXPEHU
“
*UDSKWKHWULJRQRPHWULFIXQFWLRQV\ VLQșDQG\ FRV θ − π
2
RYHUWKHLQWHUYDO²ʌ”ș”ʌRQDJUDSKLQJFDOFXODWRU:KDWGR
\RXQRWLFHDERXWWKHJUDSKV"([SODLQVHHPDUJLQ
(
)
)XQGDPHQWDO7KHRUHPRI&RXQWLQJ
9. a.(0.28)3 = 2.2%
b.1 – 0.28 = 0.72
c.0.28 • 0.28 • 0.72 = 5.6%
d.0.72 • 0.28 • 0.72 = 14.5%
Sample answer: the graphs
are the same because the sine
and cosine functions are π
2
radians out of phase.
Math Applications for this
chapter are on pages 542–549.
The notes and solutions shown
below accompany the suggested
applications to assign with this
lesson.
4. a. 725 = 29
3, 000 120
b. 725 − 1 = 724
3, 000 − 1 2, 999
c. 29 • 724 ≈ 0.058
120 2, 999
≈ 5.8%
6. a.Because each click does
not depend on any
other event, they are
independent events.
b.Because each click
is independent, the
probability remains the
same: 0.14.
c.Because each click
is independent, the
probability remains the
same: 0.14.
d.0.14 • 0.14 • 0.14 • 0.14
≈ 0.0004 ≈ 0.04%
11.1 Fundamental Theorem of Counting 513
LESSON PLANNING
/HVVRQ &RPSRXQG(YHQWV
/
Vocabulary
2EMHFWLYHV
2
)LQGWKHSUREDELOLW\RI
FRPSRXQGHYHQWV
compound event
mutually exclusive
inclusive events
conditional probability
6LJPXQGSLFNHGDSDLURIVRFNVRXWRIKLVGUDZHU
ZLWKRXWORRNLQJLQWKHGUDZHU6LJPXQGKDVSDLUVRI
EODFNGUHVVVRFNVSDLUVRIEURZQGUHVVVRFNVSDLUV
RIEODFNVSRUWVRFNVDQGSDLUVRIZKLWHVSRUWVRFNV
:KDWLVWKHSUREDELOLW\WKDW6LJPXQGZLOOVHOHFWDSDLU
RIEODFNVRFNVRUDSDLURIVSRUWVRFNV"
0XWXDOO\([FOXVLYH(YHQWV
Extra Resources
$FRPSRXQGHYHQWLQYROYHVWZRRUPRUHHYHQWVVXFKDVWRVVLQJWKUHH
QXPEHUFXEHVRUFKRRVLQJWZRSHRSOHWRVHUYHRQDFRPPLWWHH:KHQ
WZRHYHQWVFDQQRWRFFXUDWWKHVDPHWLPHWKHHYHQWVDUHPXWXDOO\
H[FOXVLYH
Reteaching 11.2
Extra Practice 11.2
Enrichment 11.2
3UREDELOLW\RI0XWXDOO\([FOXVLYH(YHQWV
Assignment
,I$DQG%DUHPXWXDOO\H[FOXVLYHHYHQWVWKHQWKH
SUREDELOLW\WKDW$RU%ZLOORFFXULVJLYHQE\WKHIRUPXOD
In-class practice: 1–5
Homework: 6–37
3$RU% 3$3%
,I$DQG%DUHPXWXDOO\H[FOXVLYHHYHQWVWKHQ3$DQG% VLQFH
$DQG%FDQQRWRFFXUDWWKHVDPHWLPH
Math Applications
([DPSOH )LQGLQJ3UREDELOLW\RI
0XWXDOO\([FOXVLYH(YHQWV
Exercises 1, 3, 7, 10, and 12 from
pages 542–549
$IUXLWEDVNHWFRQWDLQVSHDUVUHGDSSOHVJUHHQDSSOHVDQG
RUDQJHV,I.DUHQUHDFKHVLQWRWKHEDVNHWDQGVHOHFWVDSLHFHRIIUXLW
DWUDQGRPZKDWLVWKHSUREDELOLW\WKDWLWLVHLWKHUUHGRURUDQJH"
START UP
6ROXWLRQ
Demonstrate the concepts of
mutual exclusion and inclusion in
the following way. First, have all
female students sit on the left side
of the room and all male students
sit on the right side of the room.
Second, have all students sit on
the left side of the room and all
male students sit on the right side
of the room.
2QO\WKHUHGDSSOHVDUHUHGDQGRQO\WKHRUDQJHVDUHRUDQJH6RWKH
HYHQWVRISLFNLQJDUHGSLHFHRIIUXLWRUDQRUDQJHSLHFHRIIUXLWDUH
PXWXDOO\H[FOXVLYH$SLHFHRIIUXLWFDQQRWEHUHGDQGRUDQJHDWWKH
VDPHWLPH
3UHGRURUDQJH 3UHG3RUDQJH
3UHG 2
12
3RUDQJH 3
12
3UHGRURUDQJH 2 + 3 5
12 12 12
7KHSUREDELOLW\WKDWWKHSLHFHRIIUXLWLVHLWKHUUHGRURUDQJHLV 5 12
&KDSWHU3UREDELOLW\DQG6WDWLVWLFV
INSTRUCTION
Example 1 Ask students to
calculate the probability that the
fruit picked is not red or orange
once they know the answer to the
original problem.
Emphasize that P(A and B) = 0
when A and B are mutually
exclusive events by asking
students to calculate the
probability P(apple and orange).
Diversity in the Classroom
English Language Learner
Some students will struggle with new vocabulary in this lesson. Students
probably computed problems such as Example 1 in previous courses without
using the formula. Remind them that Example 1 could have been calculated in
the following way.
P(orange or red) = number of winners
number of possible
P(oraange or red) = 5
12
514 Chapter 11 Probability and Statistics
INSTRUCTION
2QJRLQJ$VVHVVPHQW
7KHUHDUHQRYHOVELRJUDSKLHVSRHWU\ERRNVDQG
VFLHQFHERRNVRQDVKHOI,I3DXODVHOHFWVDERRNDW
UDQGRPZKDWLVWKHSUREDELOLW\WKDWLWLVDELRJUDSK\RUD
SRHWU\ERRN"([SUHVV\RXUDQVZHUDVDIUDFWLRQ 1
3
,QFOXVLYH(YHQWV
(YHQWVWKDWDUHQRWPXWXDOO\H[FOXVLYHDUHLQFOXVLYHHYHQWV7KHVHDUH
HYHQWVWKDWFDQKDSSHQDWWKHVDPHWLPH)RUH[DPSOHZKHQFDOFXODWLQJWKH
SUREDELOLW\RIVHOHFWLQJDGLDPRQGRUDIDFHFDUGWKHUHDUHWKUHHPXWXDOO\
LQFOXVLYHHYHQWVSLFNLQJDMDFNRIGLDPRQGVDTXHHQRIGLDPRQGVDQGD
NLQJRIGLDPRQGV
3UREDELOLW\RI,QFOXVLYH(YHQWV
,I$DQG%DUHLQFOXVLYHHYHQWVWKHQWKHSUREDELOLW\WKDW
$RU%ZLOORFFXULVJLYHQE\WKHIRUPXOD
3$RU% 3$3%²3$DQG%
Example 2 Show students a
Venn diagram which represents
the socks in Sigmund’s drawer.
This diagram will provide students
of a visual representation of why
P(A and B) must be subtracted.
Ask students to explain why the
selection of the socks from the
drawer might not be a completely
random event.
Tell students that another term
used to describe inclusive events is
“non-mutually exclusive” events.
([DPSOH )LQGLQJ3UREDELOLW\RI,QFOXVLYH(YHQWV
$WWKHEHJLQQLQJRIWKLVOHVVRQDVLWXDWLRQZDVSUHVHQWHGLQZKLFK6LJPXQG
ZDVVHOHFWLQJVRFNVIURPDGUDZHUZLWKRXWORRNLQJ:KDWLVWKHSUREDELOLW\
WKDW6LJPXQGZLOOVHOHFWDSDLURIEODFNVRFNVRUDSDLURIVSRUWVRFNV"
6ROXWLRQ
7KHHYHQWVDUHLQFOXVLYHHYHQWVEHFDXVHDSDLURIVRFNVFDQEHERWKDVSRUW
VRFNDQGEODFN
3EODFNRUVSRUW 3EODFN3VSRUW±3EODFNDQGVSRUW
6
3EODFN 15
8
3VSRUW 15
3
3EODFNDQGVSRUW 15
11
3EODFNRUVSRUW 6 + 8 − 3 15
15 15 15
7KHSUREDELOLW\WKDW6LJPXQGVHOHFWVDSDLURIEODFNVRFNVRUDSDLURIVSRUW
VRFNVLV 11 15
&RPSRXQG(YHQWV
Enriching the Lesson
Ask students to justify why the formula for calculating
the P(A or B) when A and B are inclusive events can also
be used when calculating the P(A or B) when A and B are
mutually exclusive events. Tell students to use examples in
their justification.
11.2 Compound Events
515
INSTRUCTION
2QJRLQJ$VVHVVPHQW
Explain to students that
conditional probabilities are those
in which the sample space for
an event is restricted. Remind
students that a sample space is
the set of all possible outcomes of
a probability event.
7KHUHDUHJLUOVDQGER\VLQ0LVV5HDGLQJ¶VKRPHURRP)LYHRIWKHJLUOV
SOD\VSRUWVDQGGRQRWSOD\VSRUWV(LJKWRIWKHER\VSOD\VSRUWVDQGGR
QRWSOD\VSRUWV,IDVWXGHQWLVVHOHFWHGDWUDQGRPZKDWLVWKHSUREDELOLW\WKDW
WKHVWXGHQWLVDER\RUSOD\VVSRUWV"([SUHVV\RXUDQVZHUDVDIUDFWLRQ 17
20
&RQGLWLRQDO(YHQWV
$FRQGLWLRQDOSUREDELOLW\LVWKHSUREDELOLW\WKDWRQHHYHQWZLOORFFXUJLYHQ
WKDWDQRWKHUHYHQWKDVDOUHDG\RFFXUUHG7KLVFRQFHSWZDVLQWURGXFHGZLWK
GHSHQGHQWHYHQWV
Tell students to think of
conditional probability in terms
of deciding what kind of car you
buy. The model of car is going to
depend on the make of car that
is available at the car dealership
where you are shopping.
3UREDELOLW\RI&RQGLWLRQDO(YHQWV
7KHFRQGLWLRQDOSUREDELOLW\RIHYHQW$JLYHQHYHQW%KDV
DOUHDG\RFFXUUHGLVJLYHQE\WKHIRUPXOD
P( A and B)
3$_% ZKHUH3%
P ( B)
([DPSOH )LQGLQJ3UREDELOLW\RI&RQGLWLRQDO(YHQWV
7KHWDEOHEHORZVKRZVWKHUHVXOWVRIDFODVVVXUYH\)LQGWKHFRQGLWLRQDO
SUREDELOLW\WKDWDVWXGHQWGLGPRUHWKDQKRXUVRIKRPHZRUNODVWQLJKW
JLYHQWKDWWKHVWXGHQWLVDIHPDOH
'LG\RXGRPRUHWKDQKRXUV
RIKRPHZRUNODVWQLJKW"
0DOH
)HPDOH
<HV
1R
6ROXWLRQ
7KHUHDUHPDOHVDQGIHPDOHVIRUDWRWDORIVWXGHQWV$WRWDORI
VWXGHQWVGLGPRUHWKDQKRXUVRIKRPHZRUNZKLOHVWXGHQWVGLGQRW
3 (more than 2 hours and female)
3 (female)
10
10
32
3PRUHWKDQKRXUV_IHPDOH 18 = 18 5
9
32
7KHSUREDELOLW\WKDWDVWXGHQWGLGPRUHWKDQKRXUVRIKRPHZRUNODVWQLJKW
JLYHQWKDWWKHVWXGHQWLVDIHPDOHLV 5 9
3PRUHWKDQKRXUV_IHPDOH &KDSWHU3UREDELOLW\DQG6WDWLVWLFV
Diversity in the Classroom
Visual Learner
Help students to understand conditional probabilities better by constructing
tree diagrams. Each first branch in the tree diagram will represent a simple
probability. Each second branch in the tree diagram will represent a conditional
probability.
516 Chapter 11 Probability and Statistics
$FWLYLW\
ACTIVE LEARNING
5HFHVVLYH7UDLWV
,QWKHVWXG\RIKXPDQJHQHWLFVDGRPLQDQWJHQHLVSDUWRIDSDLU
RIWUDLWVVXFKDVH\HFRORUWKDWLVSDVVHGRQWRRIIVSULQJ7KHRWKHU
SDUWRIWKHSDLURIWUDLWVLVWKHUHFHVVLYHJHQH$GRPLQDQWJHQHZLOO
´RYHUSRZHUµDUHFHVVLYHJHQHWKDWUHSUHVHQWVWKHVDPHWUDLW
$3XQQHW6TXDUHLVDPDWKHPDWLFDOZD\WRVKRZZKDWJHQHWLFWUDLWV
RIIVSULQJPLJKWLQKHULWIURPWKHLUSDUHQWV,QKXPDQVWKHWUDLWEHLQJ
ULJKWKDQGHGLVGRPLQDQWRYHUWKHWUDLWEHLQJOHIWKDQGHG,QWKH
3XQQHW6TXDUHEHORZ5UHSUHVHQWVWKHGRPLQDQWWUDLWDQGUUHSUHVHQWV
WKHUHFHVVLYHWUDLW
5
U
5
55
5U
U
5U
UU
,QWKH3XQQHW6TXDUHERWKSDUHQWVFDUU\WKHGRPLQDQW5DVZHOO
DVWKHUHFHVVLYHU7KLVPHDQVWKDWERWKSDUHQWVDUHULJKWKDQGHG
$JHQRW\SHLVDSRVVLEOHFRPELQDWLRQRIJHQHVLQKHULWHGE\WKH
RIIVSULQJVXFKDV55RU5U+RZPDQ\JHQRW\SHVDUHVKRZQLQWKH
WDEOH"/LVWWKHP555U5UDQGUU
2IWKHJHQRW\SHVKRZPDQ\RIWKHPZLOOUHVXOWLQDQRIIVSULQJ
EHLQJULJKWKDQGHG"
2IWKHJHQRW\SHVKRZPDQ\RIWKHPZLOOUHVXOWLQDQRIIVSULQJ
EHLQJOHIWKDQGHG"
:KDWLVWKHFRQGLWLRQDOSUREDELOLW\WKDWDFKLOGZLOOEHERUQOHIW
KDQGHGJLYHQWKDWERWKSDUHQWVFDUU\WKHGRPLQDQW5DQGWKH
UHFHVVLYHU"
Discuss with students other
inherited traits and whether these
traits are dominant or recessive.
Use the Internet for research
purposes.
Calculate the experimental
probability of a student in your
class being left handed. Compare
this experimental probability
to the theoretical probability
calculated in the activity.
/HVVRQ$VVHVVPHQW
7KLQNDQG'LVFXVVVHHPDUJLQ
:KDWLVDFRPSRXQGHYHQW"*LYHDQH[DPSOH
:KDWGRHVLWPHDQIRUWZRHYHQWVWREHPXWXDOO\H[FOXVLYH"
:KDWGRHVLWPHDQIRUWZRHYHQWVWREHLQFOXVLYH"
+RZFDQ\RXGHWHUPLQHWKHSUREDELOLW\WKDWHLWKHURQHRIWZR
LQFOXVLYHHYHQWV$DQG%ZLOORFFXU"
'HVFULEHDFRQGLWLRQDOSUREDELOLW\LQ\RXURZQZRUGV
&RPSRXQG(YHQWV
Think and Discuss Answers
1. A compound event involves two or more events. Answers will vary. Sample
answer: rolling a number cube and flipping a coin.
2. The events cannot occur at the same time.
3. The events can occur at the same time.
4. P(A or B) = P(A) + P(B) – P(A and B)
5. Answers will vary. Sample answer: the probability that one event will occur
given that another event has already occurred.
11.2 Compound Events
517
WRAP UP
To ensure mastery of objectives,
students should be able to:
• Determine the probability of
compound events which are
mutually exclusive.
• Determine the probability of
compound events which are
inclusive.
• Determine the probability of
compound events which are
conditional.
3UDFWLFHDQG3UREOHP6ROYLQJ
+HDWKHUWRVVHVWZRQXPEHUFXEHV6WDWHZKHWKHURUQRWWKH
WZRHYHQWVDUHPXWXDOO\H[FOXVLYH([SODLQ
WKHVXPUROOHGLVHYHQWKHQXPEHUVDUHWKHVDPH
WKHVXPUROOHGLVRGGERWKQXPEHUVDUHRGG
WKHVXPUROOHGLVJUHDWHUWKDQERWKQXPEHUVDUHOHVVWKDQ
WKHVXPUROOHGLVDSULPHQXPEHUWKHVXPUROOHGLVJUHDWHUWKDQ
QRWPXWXDOO\H[FOXVLYHLIWKHQXPEHUVDUHWKHVDPHWKHVXPZLOOEHHYHQ
PXWXDOO\H[FOXVLYHLIERWKQXPEHUVDUHRGGWKHVXPZLOOEHHYHQ
VHHPDUJLQ
QRWPXWXDOO\H[FOXVLYHWKHVXPFRXOGEHJUHDWHUWKDQDQGDSULPHQXPEHU
$DQG%DUHPXWXDOO\H[FOXVLYHHYHQWV)LQG3$RU%
1
3$ 3% 3$ 4 3%
3
3$ 3% 3$ 3%
22
1 12
6
7 51
110
5
/DQQ\WRVVHVDVWDQGDUGQXPEHUFXEH)LQGHDFKSUREDELOLW\
([SUHVVWKHDQVZHUVDVIUDFWLRQV
Assignment
In-class practice: 1–5
Homework: 6–37
3RGGRU 21 3RGGRUJUHDWHUWKDQ
3HYHQRUSULPH 65 3RGGRUSULPH 23
3RUHYHQ 23 3OHVVWKDQRUJUHDWHUWKDQ 5
Math Applications
8VHWKHWDEOHEHORZWRILQGHDFKFRQGLWLRQDOSUREDELOLW\
([SUHVVWKHDQVZHUVDVGHFLPDOVURXQGHGWRWKHQHDUHVW
WKRXVDQGWKLIQHFHVVDU\
Exercises 1, 3, 7, 10, and 12 from
pages 542–549
Reteaching 11.2 (CRB)

NAME
CLASS
DATE
RETEACHING
14.2 COMPOU
A compound even
ND EVENTS
choosing two peo t involves two or more even
ts such as tossing
ple to serve on
time, the events
a
three number cub
are mutually excl committee. When two events
cannot occur at es or
usive.
If A and B are
the same
mut
given by the form ually exclusive events, then
the
events, then P(A ula: P(A or B) = P(A) + P(B). probability that A or B will occu
If A and B are
and B) = 0 sinc
r is
mut
e
A
uall
and B cannot occu
Events that are
y exclusive
not
r at the same time
happen at the sam mutually exclusive are inclusiv
.
e time.
e events. These
are events that
If A and B are
can
incl
the formula: P(A usive events, then the probabil
ity that A or B
or B) = P(A) +
will occur is give
P(B) – P(A and
A conditional
B).
n by
pro
event has already bability is the probability that
one event will
occurred.
occur given that
The conditional
another
probability of even
t A, given even
t B has already
the formula: P(A
occurred, is give
|B) = P( A and B)
n by
, where P(B) ≠
P( B)
0.
EXAMPLE 1
Suppose Jamal
rolls a standard
an even number
number cube. Find
or a 6.
the probability
SOLUTION
of rolling
The events are
inclusive since
rolling a 6 is an
even number.
P(rolling an even
3
number) = = 1
;
P(rolling a 6) = 1
6 2
6
P(rolling an even
number and a 6) 1
=
6
P(rolling an even
number or a 6) 1 1 1
1
= + –
2 6 6 =
EXERCISES
2
Two number cub
es
mutually exclusiv are tossed. State whether or
not the two even
1. the sum rolle e.
ts are
d is odd; the num
bers are both odd
mutually excl
usive
is less than 7; both
numbers are less
than 4
not mutually excl
is even; the num
usive
bers are both even
not mutually excl
4. the sum rolle
usive
d is 12; at least
one number is
odd
mutually excl
usive
398
2. the sum rolle
d
3. the sum rolle
d
>Algebra 2 Chap
ter Resource
Copyright © CORD
Book
518 Chapter 11 Probability and Statistics
6
%OXH(\HV
*UHHQ(\HV
%URZQ(\HV
0DOH
)HPDOH
3JUHHQH\HV_IHPDOH
3EOXHH\HV_PDOH
3EURZQH\HV_PDOH
3JUHHQRUEURZQH\HV_IHPDOH
3EOXHRUJUHHQH\HV_PDOH
3IHPDOH_EURZQH\HV
&KDSWHU3UREDELOLW\DQG6WDWLVWLFV
Enrichment 11.2 (CRB)
Extra Practice 11.2 (CRB)
7KHUHDUH'9'VYLGHRJDPHV&'VDQGYLGHRWDSHVRQ
-DLPH·VEHGURRPVKHOI,I-DLPHVHOHFWVDQLWHPDWUDQGRPIURP
WKHVKHOIZKDWLVWKHSUREDELOLW\WKDWLWLVD'9'RUYLGHRWDSH"
([SUHVVWKHDQVZHUDVDIUDFWLRQ 5

12
.DUHQ·VERRNEDJFRQWDLQVQRYHOVELRJUDSK\DQGVFLHQFH
ERRN0DQQ\·VERRNEDJFRQWDLQVPDWKERRNVFLHQFHERRNV
DQGSRHWU\ERRN(DFKVWXGHQWVHOHFWVDERRNDWUDQGRPIURP
KLVRUKHUEDJ:KDWLVWKHSUREDELOLW\WKDWHLWKHU.DUHQ·VERRNLV
DQRYHORU0DQQ\·VERRNLVDPDWKERRN"([SUHVVWKHDQVZHUDV
DGHFLPDO
$VXUYH\RIGRZQWRZQZRUNHUVUHYHDOHGWKDWIHPDOHVULGH
WKHWUDLQIHPDOHVULGHWKHEXVDQGIHPDOHVFDUSRRO7KH
VXUYH\DOVRIRXQGWKDWPDOHVULGHWKHWUDLQPDOHVULGHWKH
EXVDQGPDOHVFDUSRRO
D &RPSOHWHWKHWDEOHEHORZWRRUJDQL]HWKHVXUYH\UHVXOWV
+RZGR\RXJHWWRZRUNHDFKGD\"
0DOH
)HPDOH
7UDLQ
%XV
ND EVENTS
14.2 COMPOU
ENRICHMENT
DATE
CLASS
NAME
EXTRA PRACTICE
ND EVENTS
14.2 COMPOU
(A or B).
events. Find P P(A) = 35%, P(B) = 42%
ually exclusive
2.
A and B are mut ) = 0.15
0.7
, P(B
1. P(A) = 0.32
1 (B) = 1
,P
6
3. P(A) = 2
7
in a
time to take a turn
ner is spun one
Suppose the spin whether events A and B are
e
board game. Stat e. Then find P(A or B).
mutually exclusiv
ter than 5; B: an
grea
5. A: a number
6
sible by 4; B: a
divi
8. A: a number
1
8
3
mutually excl
0.35
0.685
42
5
8
usive;
usive;
Democrat
Supports
Issue 5
Does not
5
Support Issue
No opinion
nal
find each conditio
as
Use the table to
ress the answers
probability. Exp to the nearest
decimals rounded ssary.
thousandth, if nece
support)
3
not mutually excl
by
number divisible
t)
Book>
Reso
terport
issue | Democra
s urce
Algebra
P(sup
9.2 Chap
2
4
5
mutually excl
ber
B: an even num
t | does not
11. P(Democra
0.33
not mutually excl
even number
ber
8; B: an odd num
6. A: a 4 or an
number;
7. A: a prime
77%
, P(B) = 0.22
4. P(A) = 0.11
1
2
Copyright © CORD
&DU3RRO
DATE
CLASS
NAME
3
4
usive;
usive;
1
2
Republican
82
29
63
9
15
s issue
10. P(support
7
8
403
0.683
| Republican)
ort or
P(does not supp
12. Republican)
no opinion |
0.317
Book
ter Resource
ht © CORD
E .\OHZDQWVWRNQRZZKDWSHUFHQWRIWKHZRUNHUVVXUYH\HGDUH
OLNHO\WRFDUSRROWRZRUNHDFKGD\JLYHQWKDWWKH\DUHPDOH
:KDWW\SHRISUREDELOLW\GRHVWKLVUHSUHVHQW"
>Algebra 2 Chap
Copyrig
400
FRQGLWLRQDOSUREDELOLW\
F :KDWLVWKHSUREDELOLW\WKDWDGRZQWRZQZRUNHUULGHVWKHEXV
WRZRUNHDFKGD\JLYHQWKDWWKHZRUNHULVIHPDOH"([SUHVVWKH
DQVZHUDVDGHFLPDOURXQGHGWRWKHQHDUHVWWKRXVDQGWK
G :KDWLVWKHSUREDELOLW\WKDWDGRZQWRZQZRUNHUULGHVWKH
WUDLQRUWKHEXVWRZRUNHDFKGD\JLYHQWKDWWKHZRUNHU
LVPDOH"([SUHVVWKHDQVZHUDVDGHFLPDOURXQGHGWRWKH
QHDUHVWWKRXVDQGWK
:LOOLDP·VEDWWLQJDYHUDJHFKDQFHRIJHWWLQJDKLWLVZLWK
UXQQHUVLQVFRULQJSRVLWLRQ:KDWLVWKHSUREDELOLW\WKDW:LOOLDP
ZLOOJHWDKLWGXULQJHDFKRIKLVQH[WWZRDWWHPSWVJLYHQWKDW
UXQQHUVDUHLQVFRULQJSRVLWLRQ"5RXQGWRWKHQHDUHVWWKRXVDQGWK
&RPSRXQG(YHQWV
11.2 Compound Events
519
Answers to Math
Applications
Math Applications for this
chapter are on pages 542–549.
The notes and solutions shown
below accompany the suggested
applications to assign with this
lesson.
7KHUHDUHJLUOVDQGER\VLQ0U'DGH·VPXVLFFODVV6HYHQRI
WKHJLUOVSOD\DQLQVWUXPHQWDQGGRQRW1LQHRIWKHER\VSOD\
DQLQVWUXPHQWDQGGRQRW,IDVWXGHQWLVVHOHFWHGDWUDQGRP
ZKDWLVWKHSUREDELOLW\WKDWWKHVWXGHQWLVDER\RUSOD\VDQ
22
LQVWUXPHQW"([SUHVVWKHDQVZHUDVDIUDFWLRQ 27
:LQQHUVDWDULQJWRVVJDPHJHW
DVWXIIHGDQLPDOWR\7KHUHDUH
ZKLWHOLRQVZKLWHWLJHUV
\HOORZOLRQVDQG\HOORZWLJHUV
DPRQJWKHSUL]HV$SUL]HLV
VHOHFWHGDWUDQGRP:KDWLVWKH
SUREDELOLW\WKDWWKHSUL]HLV\HOORZ
RUDOLRQ"([SUHVVWKHDQVZHUDV
DIUDFWLRQ 79
7KHSUREDELOLW\WKDWDUDQGRPO\VHOHFWHGFROOHJHVWXGHQWLV
XQGHU\HDUVROGLVDSSUR[LPDWHO\7KHSUREDELOLW\WKDW
DUDQGRPO\VHOHFWHGFROOHJHVWXGHQWLVROGHUWKDQ\HDUVROG
LVDSSUR[LPDWHO\:KDWLVWKHSUREDELOLW\WKDWDUDQGRPO\
VHOHFWHGFROOHJHVWXGHQWLVXQGHU\HDUVROGRUROGHUWKDQ"
1. a.The options are inclusive
because he could have a
red, long-sleeved shirt.
b.There are 4 red longsleeved shirts and 23
possible shirts, so the
probability is 4 .
23
c.There are 11 long-sleeved
shirts and 5 other red
shirts, so the probability is
16 .
23
d.There are 23 shirts,
16 pants, and 25 shoes,
so there are 23 • 16 • 25
= 9,200 different ways
Marcellus can dress his
avatar.
0L[HG5HYLHZ
6ROYHHDFKHTXDWLRQLQWKHJLYHQLQWHUYDO5RXQG\RXUDQVZHUV
WRWKHQHDUHVWKXQGUHGWKUDGLDQRUWHQWKGHJUHH
3,514  • 
3,513 
3. a. 1−
 1− 12,578 
12
578
,

 

≈ 0.519 ≈ 51.9%
FRVș ”șʌ
VLQșVLQșFRVș ”șʌ
FRVș²FRVș² ”șƒƒƒƒ
²FRVș ”șʌ
$ELF\FOHPDQXIDFWXUHUPDNHVURDGELNHVDQGPRXQWDLQELNHV
(DFKW\SHRIELNHFDQEHPDGHZLWKRQHRIIRXUGLIIHUHQW
VSHHGV(DFKELNHFDQEHPDGHZLWKDKDUGVHDWRUDFXVKLRQHG
VHDW+DQGOHEDUVIRUWKHELNHVPD\EHDQJOHGIRUWRXULQJRUIRU
UDFLQJ+RZPDQ\GLIIHUHQWFRPELQDWLRQVRIELF\FOHVGRHVWKH
PDQXIDFWXUHUPDNH"
b. 3, 278 + 3,113 • 3, 278 + 3,112
12,578
12,578
≈ 0.258 ≈ 25.8%
c. 274 ≈ 0.010 ≈ 1.0%
2, 674
7. a.The events are inclusive,
so P(type B or positive) =
P(type B) + P(positive) –
P(type B and positive)
= 22 + 23 − 20 = 25
38 38 38 38
b.P(type A or negative) =
P(type A) + P(negative)
– P(type A and negative) =
16 + 15 − 13 = 18 = 9
38 38 38 38 19
&KDSWHU3UREDELOLW\DQG6WDWLVWLFV
10. a.P(no seatbelt or no license) =
P(no seatbelt) + P(no license)
– P(no seatbelt and no license)
= 0.005 + 0.056 – 0.003 =
0.058
b.P(no seatbelt and no license)
= 0.003 • 2,564 ≈ 8
520 Chapter 11 Probability and Statistics
12. a.44 + 25 + 27 + 31
= 127; P = 127
235
b. 27
235
c. 27
127
/HVVRQ &ROOHFWLQJ'DWD
2EMHFWLYHV
$QDO\]HPHWKRGV
RIJDWKHULQJ
LQIRUPDWLRQ
'HVLJQDQ
DSSURSULDWH
VWXG\WRJDWKHU
LQIRUPDWLRQ
$SKDUPDFHXWLFDOFRPSDQ\ZDQWVWRWHVWWKHHIIHFWLYHQHVVRIDQHZ
GUXJLQWUHDWLQJLQVRPQLDLQDGXOWSDWLHQWV+RZFDQWKHFRPSDQ\WHVW
VW
ZKHWKHUWKHGUXJLVHIIHFWLYHLQKHOSLQJSDWLHQWVZLWKWKHLULQVRPQLD""
6XUYH\V([SHULPHQWVDQG2EVHUYDWLRQDO6WXGLHV
V
6WDWLVWLFDOVWXGLHVDUHFRQGXFWHGWRJDWKHULQIRUPDWLRQDERXWSHRSOH
DQLPDOVRUREMHFWV)RUH[DPSOHSROOVWHUVJDWKHULQIRUPDWLRQIURP
OLNHO\YRWHUVWRSUHGLFWWKHUHVXOWVRIDQHOHFWLRQDQGPDUNHWLQJ
FRPSDQLHVFRQGXFWVWXGLHVWRJDXJHEX\HUV¶LQWHUHVWLQQHZ
SURGXFWV,QDVWDWLVWLFDOVWXG\WKHJURXSEHLQJVWXGLHGLVFDOOHGWKH
SRSXODWLRQ%HFDXVHLWLVXVXDOO\LPSUDFWLFDOWRVWXG\HDFKPHPEHU
RIDSRSXODWLRQDVDPSOHRIWKHSRSXODWLRQLVRIWHQVWXGLHGDQG
JHQHUDOL]DWLRQVDUHPDGHDERXWWKHSRSXODWLRQEDVHGRQWKHVHUHVXOWV
$VXUYH\LVDVWDWLVWLFDOVWXG\LQZKLFKPHPEHUVRIDVDPSOHDUH
DVNHGWRUHVSRQGWRDTXHVWLRQRUTXHVWLRQV,QDQH[SHULPHQWD
WUHDWPHQWLVDSSOLHGWRPHPEHUVRIDVDPSOHDQGWKHUHVSRQVHLV
PHDVXUHG:LWKH[SHULPHQWVWKHVDPSOHLVGLYLGHGLQWRWZRJURXSV
DWUHDWPHQWJURXSZKRUHFHLYHWKHWUHDWPHQWDQGDFRQWUROJURXS
ZKRGRQRWUHFHLYHWKHWUHDWPHQWRUZKRUHFHLYHDSODFHER$SODFHER
R
LVDIDOVHWUHDWPHQWZKLFKLVJLYHQVRWKDWPHPEHUVRIWKHH[SHULPHQW
W
W
GRQRWNQRZZKLFKJURXSWKH\DUHLQ,QDQREVHUYDWLRQDOVWXG\
PHPEHUVRIDVDPSOHDUHVWXGLHGEXWQRWUHDWPHQWLVDSSOLHGDQGQR
DWWHPSWLVPDGHWRLQÀXHQFHWKHUHVXOWVRIWKHVWXG\
([DPSOH &ODVVLI\LQJ6WXG\7\SHV
7HOOZKHWKHUHDFKVLWXDWLRQUHSUHVHQWVDVXUYH\DQH[SHULPHQWRUDQ
REVHUYDWLRQDOVWXG\
D 7ZRKXQGUHGDGXOWVZKRGULQNVRIWGULQNVDUHVSOLWLQWRWZR
JURXSV+DOIRIWKHSDUWLFLSDQWVUHPRYHVRIWGULQNVIURPWKHLU
GLHWV7KHRWKHUKDOIRIWKHSDUWLFLSDQWVFRQWLQXHWRGULQNVRIW
GULQNV$IWHUPRQWKVWKHZHLJKWJDLQRUORVVRIWKHSDUWLFLSDQWV
WV
LQHDFKJURXSLVFRPSDUHG
E $PDQXIDFWXULQJSODQWPDQDJHUDVNVVHYHUDOHPSOR\HHVWRUDWH
WKHLUNQRZOHGJHRIVDIHW\UHJXODWLRQVRQDVFDOHIURPWR
F $GULDQFRPSDUHVWKHJUDGHSRLQWDYHUDJHVRIVWXGHQWVZKRSOD\
\
VFKRROVSRUWVDQGVWXGHQWVZKRGRQRWSOD\VFKRROVSRUWV
&ROOHFWLQJ'DWD
11.3 Collecting Data
521
6ROXWLRQ
D 7KLVVLWXDWLRQUHSUHVHQWVDQH[SHULPHQWEHFDXVHDWUHDWPHQWJLYLQJXS
VRIWGULQNVLVEHLQJDSSOLHGWRVRPHRIWKHSDUWLFLSDQWV7KHWUHDWPHQW
JURXSLVWKRVHZKRDUHJLYLQJXSVRIWGULQNVDQGWKHFRQWUROJURXSLV
WKRVHFRQWLQXLQJWRGULQNVRIWGULQNV
E 7KLVVLWXDWLRQUHSUHVHQWVDVXUYH\EHFDXVHSDUWLFLSDQWVDUHEHLQJDVNHG
WRUHVSRQGWRDTXHVWLRQ
F 7KLVVLWXDWLRQUHSUHVHQWVDQREVHUYDWLRQDOVWXG\EHFDXVHPHPEHUVRI
DVDPSOHDUHEHLQJVWXGLHGEXWWKHUHLVQRWUHDWPHQWEHLQJDSSOLHGRU
RXWVLGHLQÀXHQFHRQWKHP
2QJRLQJ$VVHVVPHQW
5HVHDUFKHUVFDOOUHVLGHQWVDQGDVNWKHPZKHWKHURUQRWWKH\ZRXOG
VXSSRUWWKHFRQVWUXFWLRQRIDQHZKLJKZD\WKURXJKWKHLUFLW\7HOOZKHWKHU
WKLVVLWXDWLRQUHSUHVHQWVDVXUYH\DQH[SHULPHQWRUDQREVHUYDWLRQDOVWXG\
7KLVUHSUHVHQWVDVXUYH\EHFDXVHSDUWLFLSDQWVDUHEHLQJDVNHGWRUHVSRQGWRDTXHVWLRQ
,IDVWDWLVWLFDOVWXG\LVELDVHGLWGRHVQRWDFFXUDWHO\PHDVXUHWKH
FKDUDFWHULVWLFVRIWKHSRSXODWLRQ%LDVFDQEHLQWURGXFHGLQWRDVWDWLVWLFDO
VWXG\LQDQXPEHURIZD\V:KHQDVDPSOHLVUHSUHVHQWDWLYHRIWKHHQWLUH
SRSXODWLRQLWLVFDOOHGDUDQGRPVDPSOHDQGLVXQELDVHG,IDVDPSOHIDYRUV
RQHRUPRUHSDUWVRIWKHSRSXODWLRQLWLVDELDVHGVDPSOH7KHZRUGLQJRI
VXUYH\TXHVWLRQVFDQIDYRURQHSRVVLEOHUHVSRQVHRYHUDQRWKHUZKLFKPD\
OHDGWRELDVHGUHVXOWV$OVRZKHQWKHSDUWLFLSDQWVRIDQH[SHULPHQWNQRZ
ZKLFKJURXSWKH\DUHLQWKHUHVXOWVRIWKHH[SHULPHQWFDQEHELDVHG
([DPSOH ,GHQWLI\LQJ%LDV
7HOOZKHWKHURUQRWWKHUHPD\EHELDVLQHDFKRIWKHIROORZLQJVLWXDWLRQV,I
VRGHVFULEHKRZWKHVWXG\FRXOGEHFKDQJHGWRUHPRYHWKHELDV
D $PDUNHWLQJFRPSDQ\VXUYH\VSHRSOHDWDJROIFRXUVHDQGDVNV
WKHPWRQDPHWKHLUIDYRULWHVSRUWWRZDWFKRQ79
E $UHVHDUFKHUFDOOVUDQGRPO\VHOHFWHGOLNHO\YRWHUVDQGDVNVWKHP
³$UH\RXH[FLWHGDERXWWKHEHQH¿WVWKDW,VVXHFRXOGEULQJWRWKH
FRPPXQLW\LILWLVSDVVHG"´
F $FRPSDQ\WKDWPDNHVKHUEDOWHD¿QGVYROXQWHHUVZKRVXIIHUIURP
UHJXODUKHDGDFKHV+DOIRIWKHSDUWLFLSDQWVGULQNWKHKHUEDOWHDHDFK
GD\DQGWKHRWKHUKDOIGRQRW$IWHUZHHNVERWKJURXSVUHSRUWRQ
ZKHWKHUWKH\KDYHVHHQDQ\LPSURYHPHQWLQWKHLUV\PSWRPV
&KDSWHU3UREDELOLW\DQG6WDWLVWLFV
522 Chapter 11 Probability and Statistics
6ROXWLRQ
D 7KLVVXUYH\PD\OHDGWRDELDVHGVDPSOHEHFDXVHSHRSOHDWDJROI
FRXUVHPD\EHPRUHOLNHO\WRSUHIHUZDWFKLQJJROIRQ797KH
PDUNHWLQJFRPSDQ\FRXOGVXUYH\SHRSOHDWDVKRSSLQJFHQWHURU
FDOOUDQGRPO\VHOHFWHGSHRSOHIURPDSKRQHGLUHFWRU\WRJHWDPRUH
UHSUHVHQWDWLYHVDPSOH
E 7KHVXUYH\TXHVWLRQLQWURGXFHVELDVE\XVLQJWKHWHUPH[FLWHGDQGE\
LPSO\LQJWKDW,VVXHZLOOEULQJEHQH¿WVWRWKHFRPPXQLW\LILWLVSDVVHG
$EHWWHUTXHVWLRQZRXOGEH³$UH\RXLQIDYRURIRURSSRVHGWR,VVXH"´
Critical Thinking
Answer
Sample answer: Those
participants who know that they
are receiving the treatment might
be inclined to imagine that their
symptoms are improving because
they expect them to improve.
F 7KLVH[SHULPHQWLVELDVHGEHFDXVHWKHSDUWLFLSDQWVDOONQRZZKLFK
JURXSWKH\DUHLQ,IWKHFRQWUROJURXSLVJLYHQDQRWKHUW\SHRIQRQ
KHUEDOWHDDVDSODFHERWKHSRWHQWLDOELDVZRXOGEHUHPRYHG
&ULWLFDO7KLQNLQJ,QSDUWFRI([DPSOHH[SODLQZK\WKHUHVXOWVRIWKH
H[SHULPHQWPLJKWEHELDVHGE\DOORZLQJWKHSDUWLFLSDQWVWRNQRZZKLFK
JURXSWKH\DUHLQVHHPDUJLQ
'HVLJQD6WDWLVWLFDO6WXG\
'HSHQGLQJRQWKHLQIRUPDWLRQVRXJKWDVXUYH\H[SHULPHQWRU
REVHUYDWLRQDOVWXG\PLJKWEHPRVWDSSURSULDWH
([DPSOH 'HVLJQLQJD6WXG\
5HIHUWRWKHLQIRUPDWLRQJLYHQLQWKHRSHQLQJRIWKHOHVVRQDQGDQVZHUHDFK
TXHVWLRQWRGHVLJQDVXLWDEOHVWXG\IRUWKHFRPSDQ\
D ,GHQWLI\WKHSRSXODWLRQDQGWKHVDPSOHIRUWKLVVLWXDWLRQ:KDW
TXHVWLRQGRHVWKHSKDUPDFHXWLFDOFRPSDQ\ZDQWWRDQVZHU"
E :RXOGDVXUYH\DQH[SHULPHQWRUDQREVHUYDWLRQDOVWXG\EHPRVW
DSSURSULDWHIRUWKHVLWXDWLRQ"([SODLQ
F 'HVFULEHWKHPHWKRGRORJ\RIWKHVWXG\WKDWWKHFRPSDQ\FRXOGXVH
6ROXWLRQ
D 7KHSRSXODWLRQLVDOODGXOWVZKRVXIIHUIURPLQVRPQLD7KHVDPSOHLV
WKHJURXSRIDGXOWVZKRZLOOEHVWXGLHG7KHSKDUPDFHXWLFDOFRPSDQ\
ZDQWVWRGHWHUPLQHZKHWKHURUQRWWKHLUQHZGUXJHIIHFWLYHO\KHOSV
DGXOWVZKRVXIIHUIURPLQVRPQLD
E $QH[SHULPHQWZRXOGEHPRVWDSSURSULDWHEHFDXVHWKHFRPSDQ\ZLOO
QHHGWRWUHDWVRPHRIWKHPHPEHUVRIWKHVDPSOHZLWKWKHQHZGUXJ
F 7KHFRPSDQ\FRXOG¿QGDGXOWYROXQWHHUVZKRVXIIHUIURP
&ROOHFWLQJ'DWD
11.3 Collecting Data
523
Ongoing Assessment
Answer
LQVRPQLDJLYHKDOIRIWKHPWKHGUXJDQGJLYHDSODFHERWRWKHRWKHU
KDOI$IWHUVHYHUDOZHHNVLQWHUYLHZHDFKSDUWLFLSDQWDQGDVNWKHP
ZKHWKHUWKH\KDYHVHHQDQ\LPSURYHPHQWLQWKHLUV\PSWRPV&RPSDUH
WKHUHVXOWVRIWKHWZRJURXSVWRVHHLIWKHQHZGUXJDSSHDUVWREH
HIIHFWLYHDWWUHDWLQJLQVRPQLD
Sample answer: As long as the
sample is randomly selected,
there should not be any bias
because participants do not know
if they are in the treatment group
or control group.
2QJRLQJ$VVHVVPHQW
,VWKHUHOLNHO\WREHDQ\ELDVLQWURGXFHGLQWRWKHVWXG\LILWLVDGPLQLVWHUHGDV
VWDWHGLQ([DPSOHSDUWF"([SODLQVHHPDUJLQ
$FWLYLW\
Activity Answers
:KHQWKHUHLVDUHODWLRQVKLSEHWZHHQWZRHYHQWVWKHUHLVFRUUHODWLRQ
EHWZHHQWKHP,IRQHRIWKHHYHQWVFDQEHVKRZQWRKDYHFDXVHG
WKHRWKHUHYHQWWKHUHLVFDXVDWLRQ:KHQDVXUYH\H[SHULPHQWRU
REVHUYDWLRQDOVWXG\LVFRQGXFWHGLWPD\EHHDV\WRVHHDFRUUHODWLRQ
EHWZHHQWZRHYHQWV+RZHYHUFDXVDWLRQFDQEHVKRZQRQO\LIWKHUHDUH
QRRWKHUDOWHUQDWLYHVWKDWFRXOGKDYHFDXVHGWKHUHODWLRQVKLS
1. Sample answer: Correlation;
While studying more would
likely improve grades, there
could be other factors
involved. For example, those
students might have gotten
better sleep the night before,
have been better test takers,
or have been more relaxed.
'HWHUPLQHZKHWKHUHDFKRIWKHIROORZLQJVLWXDWLRQVUHSUHVHQWV
FRUUHODWLRQRUFDXVDWLRQ([SODLQ\RXUUHDVRQLQJ
2. Sample answer: Correlation;
While exercising would likely
help people lose weight,
there could be other factors
involved. For example, those
in the program might have
been eating healthier.
1LNNLDVNVWKHVWXGHQWVLQKHUFODVVKRZPXFKWLPHWKH\VSHQW
VWXG\LQJIRUILQDOH[DPV6KHFRQFOXGHVWKDWVWXGHQWVZKR
VWXGLHGPRUHHDUQHGEHWWHUJUDGHVVHHPDUJLQ
,QDQH[SHULPHQWRISDUWLFLSDQWVLQDQH[HUFLVHSURJUDPORVW
ZHLJKWZKLOHRQO\RIVWXG\SDUWLFLSDQWVZKRZHUHQRWLQWKH
SURJUDPORVWZHLJKWVHHPDUJLQ
0DOFROPREVHUYHGWKDWZKHQWKHZDWHUWHPSHUDWXUHRIWKHODNH
GURSVEHORZƒ)WKHODNHEHJLQVWRIUHH]HVHHPDUJLQ
7KLQNDQG'LVFXVVVHHPDUJLQ
Think and Discuss
Answers
2. Answers will vary. Sample
answer: Are you fed up
with the incompetence of
our country’s leadership?
(biased); How many times per
day do you brush your teeth?
(unbiased)
/HVVRQ$VVHVVPHQW
3. Sample answer: Causation;
There are no other possible
alternatives that could cause
the lake water to freeze other
than the water temperature
reaching the freezing point.
1. Answers will vary. Sample
answer: See if there is a
relationship between the
number of years of service of
teachers and their students’
scores on standardized tests.
&RUUHODWLRQDQG&DXVDWLRQ
*LYHDQH[DPSOHRIDUHDOZRUOGVLWXDWLRQLQZKLFK\RXPLJKW
ZDQWWRFRQGXFWDQREVHUYDWLRQDOVWXG\
*LYHDQH[DPSOHRIDELDVHGVXUYH\TXHVWLRQDQGDQH[DPSOHRI
DQXQELDVHGVXUYH\TXHVWLRQ
([SODLQKRZWRFRQGXFWDQXQELDVHGH[SHULPHQW
'HVFULEHZKDWDUDQGRPVDPSOHPHDQVLQ\RXURZQZRUGV
&KDSWHU3UREDELOLW\DQG6WDWLVWLFV
3. Sample answer: Divide participants into 2 groups. Perform a treatment on
half of the participants and give a placebo to the other half so that they
do not know which group they are in. Compare the results of the two
groups to see if the treatment had an effect.
4. Sample answer: A random sample is a sample of members of a population
that is representative of the entire population. No part of the population is
more represented than any other part.
5. Sample answer: Correlation does not imply causation. Two events can
show a relationship without one event having caused the other. Causation
does imply correlation because if one event causes another event, the
events are automatically related to each other.
524 Chapter 11 Probability and Statistics
'RHVFRUUHODWLRQLPSO\FDXVDWLRQEHWZHHQWZRHYHQWV"'RHV
FDXVDWLRQLPSO\FRUUHODWLRQ"([SODLQ
3UDFWLFHDQG3UREOHP6ROYLQJVHHPDUJLQ
7HOOZKHWKHUHDFKVLWXDWLRQUHSUHVHQWVDVXUYH\DQH[SHULPHQW
RUDQREVHUYDWLRQDOVWXG\([SODLQ\RXUUHDVRQLQJ
$UHVHDUFKHUILQGVSDUHQWVRIJUDGHVFKRROVWXGHQWVKDOIRI
ZKRPKDYHDVWD\DWKRPHSDUHQWDQGFRPSDUHVWKHJUDGHVRI
WKHVWXGHQWV
$WHDFKHUSOD\VOLJKWFODVVLFDOPXVLFGXULQJWHVWWDNLQJIRUKLVILUVW
SHULRGFODVVEXWGRHVQRWSOD\PXVLFGXULQJWHVWWDNLQJIRUKLV
VHFRQGSHULRGFODVV+HFRPSDUHVWKHWHVWVFRUHVRIWKHWZRFODVVHV
$UHVWDXUDQWRZQHUZDQWVWRNQRZZKLFKRIQHZHQWUHHV
KLVFXVWRPHUVZRXOGOLNHWRVHHRIIHUHGRQWKHPHQX+HDVNV
FXVWRPHUVDWWKHLUWDEOHVWRJLYHWKHLURSLQLRQVGXULQJGLQQHUVHUYLFH
7HOOZKHWKHUHDFKVXUYH\TXHVWLRQLVELDVHGRUXQELDVHG
+RZRIWHQGRHV\RXUIDPLO\HDWRXWDWUHVWDXUDQWVHDFKPRQWK"
'R\RXSUHIHUZDWFKLQJWKULOOLQJDFWLRQPRYLHVRUFRPHGLHV"
7HOOZKHWKHUHDFKVDPSOHLVOLNHO\WREHELDVHGRUUDQGRP
(PSOR\HHVDWDQDXWRPRELOHPDQXIDFWXULQJSODQWDUHDVNHGWR
QDPHWKHLUIDYRULWHPDNHRIFDU
$FLW\ERDUGPHPEHUPDLOVDVXUYH\WRHYHU\FLW\UHVLGHQWZKRVH
PDLOLQJDGGUHVVHQGVLQDUDQGRPO\FKRVHQGLJLWIURPWKURXJK
'HVFULEHDSRVVLEO\VWXG\IRUHDFKRIWKHIROORZLQJVLWXDWLRQV
)RUHDFKVLWXDWLRQ
D ,GHQWLI\WKHSRSXODWLRQWKHVDPSOHDQGWKHTXHVWLRQWKDW
LVWREHDQVZHUHG
E 'HWHUPLQHZKHWKHUDVXUYH\H[SHULPHQWRUREVHUYDWLRQDO
VWXG\LVPRVWDSSURSULDWH
F 'HVFULEHWKHPHWKRGRORJ\RI\RXUVWXG\DQGZKHWKHULW
ZRXOGJLYHELDVHGRUXQELDVHGUHVXOWV
.DUHQZDQWVWRGHWHUPLQHZKHWKHURUQRWVWXGHQWVLQKHUVFKRRO
IDYRUDVFKRROGUHVVFRGH
$JDUGHQLQJFRPSDQ\ZDQWVWRVHHLIWKHLUQHZSODQWIRRGKDVDQ
LQFUHDVHGHIIHFWRQWKHJURZWKRIIORZHULQJSODQWV
&ROOHFWLQJ'DWD
all flowering plants, and the sample is the flowering plants to be studied.
The company wants to know if using their plant food has an appreciable
effect on the growth of flowering plants. (b) An experiment would be
most appropriate because the company will treat some of the plants with
their food. (c) The company can take a sample of 100 flowering plants
and divide them into 2 groups. Treat one half of the plants with the new
plant food and treat the other half of the plants with a regular plant food.
Give each plant the same amount of water and sunlight so that growing
conditions are identical. Measure the growth of the plants in each group
after several weeks and compare the results. The experiment should be
unbiased since a plant could be assigned to either group and will receive
the exact same growing conditions.
Practice and Problem
Solving Answers
6. Observational study; no
treatment applied to either
group of parents; no attempt
to influence the results.
7. Experiment; teacher is
applying a treatment (playing
music during test taking) for
one group but not the other.
8. Survey; a question is being
asked to gather information.
9. Unbiased; does not favor a
particular response.
10. Biased; the word thrilling
could sway the respondent to
favor action movies.
11. Likely biased; employees of a
certain make of car might be
inclined to say that it is their
favorite make.
12. Likely unbiased; each city
resident has an equal chance
of being mailed the survey.
13. Answers will vary. Sample
answer: (a) The population
is all students at Karen’s
school, and the sample is
the students whose opinions
Karen will gather. She wants
to determine whether the
students favor a school dress
code. (b) A survey would be
most appropriate because
Karen wants to gather
opinions on an issue. (c)
Karen can select a random
sample of 50 students and
ask them, “Do you support or
oppose having a school dress
code?” The survey results
should be unbiased because
the sample is representative
of the population and the
survey question is unbiased.
14. Answers will vary. Sample
answer: (a) The population is
11.3 Collecting Data
525
/HVVRQ 3UREDELOLW\0RGHOV
/
2EMHFWLYHV
2
8VHSUREDELOLW\WRPRGHO
UDQGRPHYHQWV
8VHH[SHFWHGYDOXHWR
PDNHGHFLVLRQV
$IDVWIRRGUHVWDXUDQWLQFOXGHVRI
GLIIHUHQWSUL]HVLQHDFKRIWKHLUNLGV¶
PHDOV+RZPDQ\NLGV¶PHDOVZRXOG
\RXSUHGLFWDFXVWRPHUQHHGVWREX\
RQDYHUDJHLQRUGHUWRJHWDOOSUL]HV"
0RGHOLQJ5DQGRPQHVV
5DQGRPHYHQWVDUHHYHQWVWKDWKDYHQRSUHGHWHUPLQHGRXWFRPHRU
ELDVWRZDUGDSDUWLFXODURXWFRPH(DFKSRVVLEOHRXWFRPHLVHTXDOO\
OLNHO\WRRFFXU)RUH[DPSOHZKHQ\RXUROODQXPEHUFXEHWKHYDOXH
RIWKHUROOUHSUHVHQWVDUDQGRPHYHQWEHFDXVHHDFKQXPEHULVHTXDOO\
OLNHO\WREHUROOHG
3UREDELOLW\SOD\VDQLPSRUWDQWUROHLQPRGHOLQJUDQGRPQHVVDQG
GHFLVLRQPDNLQJ,QRUGHUIRUDQHYHQWWREHWUXO\UDQGRPHDFK
SRVVLEOHRXWFRPHPXVWKDYHWKHVDPHSUREDELOLW\6LPLODUO\DIDLU
GHFLVLRQLVDGHFLVLRQWKDWGRHVQRWVKRZDQ\ELDVRUIDYRUWRZDUGDQ\
RIWKHSRVVLEOHFKRLFHV
([DPSOH 8VLQJD5DQGRP1XPEHU*HQHUDWRU
7KHUHDUHVWXGHQWVLQ0U-RKQVWRQ¶VPDWKFODVV+HZDQWVWRVHOHFW
RIWKHVWXGHQWVWRFRPHWRWKHERDUGDQGSUHVHQWWKHLUVROXWLRQVWRD
SUREOHP'HVFULEHKRZ0U-RKQVWRQFDQXVHUDQGRPQXPEHUVWRKHOS
KLPPDNHDIDLUGHFLVLRQ
6ROXWLRQ
7KHUHDUHVHYHUDOZD\V0U-RKQVWRQFRXOGPDNHDIDLUGHFLVLRQDERXW
ZKLFKVWXGHQWVZLOOFRPHWRWKHERDUG)RUH[DPSOHKHFRXOGZULWH
DOORIWKHVWXGHQWV¶QDPHVRQVOLSVRISDSHUPL[WKHPXSLQDKDWDQG
SXOORXWQDPHVZLWKRXWORRNLQJ
PLACEHOLDER
$VLPSOHUZD\WRPDNHDIDLUGHFLVLRQLVWRXVHWKH
UDQG,QWIXQFWLRQRQDJUDSKLQJFDOFXODWRU0U-RKQVWRQ
FDQDVVLJQHDFKVWXGHQWDQXPEHUIURPWKURXJKDQG
XVHWKHIXQFWLRQFDOOUDQG,QWWRJHQHUDWHUDQGRP
LQWHJHUVIURPWRRQKLVFDOFXODWRU
7KHVWXGHQWVZKRZHUHDVVLJQHGQXPEHUVDQGZLOO
FRPHWRWKHERDUG
&KDSWHU3UREDELOLW\DQG6WDWLVWLFV
526 Chapter 11 Probability and Statistics
2QJRLQJ$VVHVVPHQWVHHPDUJLQ
:KDWLVDQRWKHUZD\WKDW0U-RKQVWRQFRXOGKDYHPDGHWKHIDLUGHFLVLRQ"
$SUREDELOLW\PRGHOLVPDWKHPDWLFDOUHSUHVHQWDWLRQRIDUDQGRPHYHQW
WKDWXVHVSUREDELOLWLHVWRGHVFULEHWKHSRVVLEOHRXWFRPHVRIWKHHYHQW$
VLPXODWLRQLVDW\SHRISUREDELOLW\PRGHOWKDWFDQEHXVHGWRUHFUHDWHDQ
HYHQWRIWHQUHSHDWHGO\WRPDNHSUHGLFWLRQVDERXWWKHRXWFRPHVRIWKHHYHQW
&RQGXFWLQJD6LPXODWLRQ
Ongoing Assessment
Answers
Answers will vary. Sample answer:
Choose 3 dates at random
and have the students whose
birthdays are closest to those
dates come to the board.
7KHUHDUHNH\VWHSVWRFRQGXFWLQJDVLPXODWLRQ
,GHQWLI\DOORIWKHSRVVLEOHRXWFRPHVRIWKHHYHQW
DQGGHWHUPLQHWKHSUREDELOLW\RIHDFKRXWFRPH
'HVLJQDQDSSURSULDWHSUREDELOLW\PRGHOIRUWKH
HYHQWEDVHGRQWKHVHSUREDELOLWLHV
'HWHUPLQHZKDWHDFKWULDORIWKHVLPXODWLRQ
UHSUHVHQWVFRQGXFWVHYHUDOWULDOVRIWKHVLPXODWLRQ
DQGUHFRUGWKHUHVXOWV$QDO\]HWKHUHVXOWV
([DPSOH &RQGXFWLQJD6LPXODWLRQ
5HIHUWRWKHVFHQDULRDWWKH
EHJLQQLQJRIWKHOHVVRQ
'HVLJQDQGFRQGXFWD
VLPXODWLRQWRSUHGLFWKRZ
PDQ\NLGV¶PHDOVRQDYHUDJH
DFXVWRPHUQHHGVWREX\LQ
RUGHUWRJHWDOOSUL]HV
6ROXWLRQ
)ROORZWKHNH\VWHSVLQWKHEOXHER[WRVHWXSDQGFRQGXFWDVLPXODWLRQ
6WHS 7KHUHDUHSRVVLEOHRXWFRPHVSUL]HSUL]HSUL]HSUL]H
SUL]HDQGSUL]H$VVXPLQJHDFKSUL]HLVHTXDOO\OLNHO\WREH
1
LQFOXGHGLQDNLGV¶PHDOWKHSUREDELOLW\RIJHWWLQJHDFKSUL]HLV 6
6WHS 7KLQNRIDSUREDELOLW\PRGHOWKDWKDVHTXDOO\OLNHO\RXWFRPHV
<RXFDQUROODQXPEHUFXEHDQGOHWWKHQXPEHURIHDFKUROO
UHSUHVHQWHDFKRIWKHSUL]HV
3UREDELOLW\0RGHOV
11.4 Probability Models
527
Critical Thinking
Answer
Sample answer: The simulation
would likely give a more accurate
prediction if more trials are
conducted.
6WHS 5ROOWKHQXPEHUFXEHDQGNHHSWUDFNRIWKHUHVXOWVLQDIUHTXHQF\
WDEOH&RQWLQXHWKHVLPXODWLRQXQWLO\RXKDYHUROOHGHDFKQXPEHU
DWOHDVWRQFHEXWVWRSDVVRRQDVDOOQXPEHUVKDYHEHHQUROOHG
5ROO3UL]H1XPEHU
7DOO\
____
_
__
____
__
___
)UHTXHQF\
7KLVFRPSOHWHVWULDORIWKHVLPXODWLRQ7KHUHVXOWVRIWKLVWULDOVKRZWKDW
LWZRXOGWDNH NLGV¶PHDOVLQRUGHUWRJHWDOO
SUL]HV6XSSRVHWKHVLPXODWLRQLVFRQGXFWHGPRUHWLPHVZLWKWKHIROORZLQJ
UHVXOWVDQG
%DVHGRQWKHVHWULDOVRIWKHVLPXODWLRQDFXVWRPHUZRXOGQHHGWREX\
· NLGV¶PHDOVLQRUGHUWRJHWDOOSUL]HVRQDYHUDJH
&ULWLFDO7KLQNLQJ+RZDFFXUDWHGR\RXWKLQNWKLVSUHGLFWLRQLV"+RZ
FRXOG\RXLPSURYHRQWKHDFFXUDF\RIWKHSUHGLFWLRQ"VHHPDUJLQ
([SHFWHG9DOXHDQG'HFLVLRQ0DNLQJ
,Q([DPSOHDVLPXODWLRQZDVFRQGXFWHGWRSUHGLFWKRZPDQ\NLGV¶
PHDOVZRXOGQHHGWREHSXUFKDVHGLQRUGHUWRJHWDOOSUL]HV$FXVWRPHU
FRXOGXVHWKLVLQIRUPDWLRQWRGHFLGHZKHWKHURUQRWLWLVZRUWKWKHLUWLPH
HIIRUWDQGPRQH\WRWU\WRFROOHFWDOOSUL]HV([SHFWHGYDOXHDOVRXVHV
SUREDELOLW\WRKHOS\RXSUHGLFWUHVXOWVDQGPDNHGHFLVLRQV
([SHFWHG9DOXH
7RILQGWKHH[SHFWHGYDOXHRIDQHYHQWPXOWLSO\WKH
SUREDELOLW\RIHDFKRXWFRPHE\WKHQXPHULFDOYDOXHRI
HDFKRXWFRPHDQGDGGWKHUHVXOWV
/HW$EHDQHYHQWZLWKRXWFRPHV$$$«WKDWHDFK
KDYHDQXPHULFDOYDOXH9$9$9$DQGVRRQ7KH
H[SHFWHGYDOXHRIHYHQW$GHQRWHG($LVJLYHQE\
($ 3$ă9$3$ă9$3$ă9$«
&KDSWHU3UREDELOLW\DQG6WDWLVWLFV
528 Chapter 11 Probability and Statistics
([DPSOH &DOFXODWLQJ([SHFWHG9DOXH
%DVHGRQSUHYLRXVUHVXOWVWKH7LJHUVIRRWEDOOWHDPKDVDQFKDQFHRI
VFRULQJDSRLQWWRXFKGRZQRQWKHLURSHQLQJGULYHRIDJDPHDQGD
FKDQFHRIVFRULQJDSRLQW¿HOGJRDO2WKHUZLVHWKH\ZLOOQRWVFRUHDQ\
SRLQWVRQWKHLURSHQLQJGULYH)LQGWKHH[SHFWHGYDOXHRIWKHQXPEHURI
SRLQWVWKH7LJHUVZLOOVFRUHRQWKHLUQH[WRSHQLQJGULYH
6ROXWLRQ
/LVWWKHQXPHULFDOYDOXHRIHDFKSRVVLEOHRXWFRPHDQGLWVDVVRFLDWHG
SUREDELOLW\
2XWFRPH
7RXFKGRZQ
)LHOG*RDO
1R6FRUH
1XPHULFDO9DOXH
SRLQWV
SRLQWV
SRLQWV
3UREDELOLW\
±± 0XOWLSO\WKHSUREDELOLW\RIHDFKRXWFRPHE\WKHFRUUHVSRQGLQJQXPHULFDO
YDOXHDQGDGGWR¿QGWKHH[SHFWHGYDOXH
(SRLQWVVFRUHGRQRSHQLQJGULYH 7KHH[SHFWHGQXPEHURISRLQWVVFRUHGRQWKHRSHQLQJGULYHLVSRLQWV
2QJRLQJ$VVHVVPHQW
-DQLHKDVQLFNHOVGLPHVDQGTXDUWHUVLQKHUFKDQJHSXUVH,IVKH
VHOHFWVDFRLQDWUDQGRPZKDWLVWKHH[SHFWHGYDOXHRIWKHFRLQ"
([DPSOH 8VLQJ([SHFWHG9DOXHWR0DNH'HFLVLRQV
&DUOWRQLVFRQVLGHULQJLQYHVWLQJLQDFRPSDQ\$QDO\VWVSUHGLFWWKDW
WKHUHLVDFKDQFHWKDWLQYHVWRUVZLOOWULSOHWKHLUPRQH\DFKDQFH
WKDWWKH\ZLOOGRXEOHWKHLUPRQH\DQGDFKDQFHWKDWWKH\ZLOOORVHWKHLU
LQYHVWPHQW6KRXOG&DUOWRQLQYHVWLQWKHFRPSDQ\"
6ROXWLRQ
)LQGWKHH[SHFWHGYDOXHRIWKHUHWXUQRQ&DUOWRQ¶VLQYHVWPHQW
(UHWXUQRQLQYHVWPHQW ±
±
%HFDXVHWKHH[SHFWHGYDOXHRIWKHUHWXUQRQ&DUOWRQ¶VLQYHVWPHQWLVDJDLQRI
KHVKRXOGLQYHVWLQWKHFRPSDQ\
3UREDELOLW\0RGHOV
11.4 Probability Models
529
Ongoing Assessment
Answer
2QJRLQJ$VVHVVPHQW
0DUWLQDFDQDWWHPSWWRPDNHDSRLQWEDVNHWRUSDVVWKHEDOOWRDWHDPPDWH
ZKRZLOODWWHPSWDSRLQWEDVNHW,I0DUWLQDKDVDFKDQFHRIPDNLQJ
KHUVKRWDQGWKHWHDPPDWHKDVDFKDQFHRIPDNLQJWKHSRLQWVKRW
ZKDWVKRXOG0DUWLQDGR"([SODLQVHHPDUJLQ
She should pass the ball to the
teammate because the expected
value of her shot is 1.2 points,
and the expected value of her
teammate’s shot is 1.4 points.
/HVVRQ$VVHVVPHQW
7KLQNDQG'LVFXVVVHHPDUJLQ
Think and Discuss
Answers
1. Answers will vary. Sample
answer: Random numbers
can be used to represent each
outcome of an event with no
bias toward any particular
outcome. For example, you
can use random numbers to
decide who has to wash the
dishes after dinner.
'HVFULEHKRZ\RXFDQXVHUDQGRPQXPEHUVWRPDNHDIDLU
GHFLVLRQ*LYHDQH[DPSOH
([SODLQZKDWLWPHDQVIRUDQHYHQWWREHUDQGRPLQ\RXURZQZRUGV
'HVFULEHWKHDGYDQWDJHVRIFRQGXFWLQJDVLPXODWLRQDQGKRZD
VLPXODWLRQFDQEHXVHGWRKHOS\RXPDNHDSUHGLFWLRQ
:LOOWKHH[SHFWHGYDOXHRIDQHYHQWEHWKHVDPHDVWKHDFWXDO
YDOXH"([SODLQ
([SODLQKRZFDOFXODWLQJH[SHFWHGYDOXHFDQKHOS\RXPDNHD
GHFLVLRQ
3UDFWLFHDQG3UREOHP6ROYLQJVHHPDUJLQ
'HVFULEHKRZ\RXFDQXVHUDQGRPQHVVWRPDNHDIDLUGHFLVLRQ
LQHDFKVLWXDWLRQ
2. Answers will vary. Sample
answer: For an event to be
random it means that there
is no bias or leaning toward
any particular outcome. Each
outcome is equally likely to
occur.
3. Answers will vary. Sample
answer: A simulation can
allow you to see what is likely
to happen in an experiment
without actually conducting
the experiment. It may be
costly or impractical to
conduct the experiment. The
results of the simulation can
give you insight into what
will likely happen. You can
use this information to make
predictions about the event.
7HUUHOO&DUPHQDQG$OLFLDDUHWU\LQJWRGHFLGHZKRJHWVWRVLWLQ
WKHIURQWVHDWRQDFDUWULS
7KHUHDUHVDOHVSHRSOHRQDVDOHVWHDP7KHPDQDJHUQHHGVWR
FKRRVHVDOHVSHRSOHWRDWWHQGDWUDLQLQJVHVVLRQ
7KHUHDUHSOD\HUVRQDIRRWEDOOWHDP7KHFRDFKQHHGVWR
FKRRVHSOD\HUVWREHFHUHPRQLDOFDSWDLQVIRUWKHFRLQWRVV
EHIRUHWKHJDPH
'HVFULEHKRZ\RXFRXOGVHWXSDQGFRQGXFWDVLPXODWLRQIRU
HDFKVLWXDWLRQ
$KRPHLPSURYHPHQWVWRUHJLYHVRXWVFUDWFKRIIFDUGVZLWKHDFK
SXUFKDVH7KHVFUDWFKRIIFDUGVHDFKFRQWDLQRIGLIIHUHQW
VSHFLDORIIHUVRQIXWXUHSXUFKDVHV2QDYHUDJHKRZPDQ\
SXUFKDVHVZRXOGDFXVWRPHUQHHGWRPDNHEHIRUHKHRUVKHJHWV
HDFKVSHFLDORIIHU"
&KDSWHU3UREDELOLW\DQG6WDWLVWLFV
4. Answers will vary. Sample answer: The expected value is not likely to be
the same as the actual value of an event, but it is possible that they are the
same. If the same experiment is conducted many times, the average results
will get closer and closer to the expected value.
5. Answers will vary. Sample answer: Calculating expected value allows you
to predict what is likely to happen in an event based on probability. You
can use this information to decide what to do in situations.
530 Chapter 11 Probability and Statistics
&ODULVVHKDVDFKDQFHRIVHUYLQJDQDFHLQYROOH\EDOO2Q
DYHUDJHKRZPDQ\VHUYHDWWHPSWVZLOO&ODULVVHQHHGEHIRUHVKH
VHUYHVDFHVLQDURZ"
&DOFXODWHHDFKH[SHFWHGYDOXH
$UDIIOHWLFNHWDWDFKXUFKIHVWLYDOKDVDFKDQFHRIEHLQJ
ZRUWKDFKDQFHRIEHLQJZRUWKDQGDFKDQFH
RIEHLQJZRUWKQRWKLQJ
7KHUHDUHWZHOYHELOOVWKUHHELOOVDQGILYHELOOVLQ
-DVRQ·VZDOOHW+HVHOHFWVRQHRIWKHELOOVZLWKRXWORRNLQJ
8VHH[SHFWHGYDOXHWRPDNHDGHFLVLRQLQHDFKVLWXDWLRQ
$VNDWHERDUGFRPSDQ\LVGHFLGLQJZKHWKHUWRLQYHVWLQDQHZ
VNDWHSDUN7KHUHLVDFKDQFHWKDWWKHFRPSDQ\ZLOOORVH
DFKDQFHWKDWWKHFRPSDQ\ZLOOEUHDNHYHQDQGD
FKDQFHWKDWWKHFRPSDQ\ZLOOHDUQRQWKHYHQWXUH
6KRXOGWKHFRPSDQ\LQYHVWLQWKHVNDWHSDUN"([SODLQKRZ\RX
PDGH\RXUGHFLVLRQ
$PDQXIDFWXULQJSODQWRZQHULVGHFLGLQJZKLFKRIWZRQHZ
SURGXFWVWRGHYHORS7KHWDEOHVKRZVWKHGHWDLOVRIHDFKSURGXFW
3URGXFW$
3URGXFW%
'HYHORSPHQW&RVW
&KDQFHRI6XFFHVV
$PRXQW(DUQHG,I6XFFHVVIXO
:KLFKRIWKHSURGXFWVVKRXOGWKHSODQWRZQHUGHYHORS"([SODLQ
KRZ\RXIRXQG\RXUDQVZHU
3UREDELOLW\0RGHOV
11.–$0.50
12.$3.85
13. Sample answer: No, the company should not invest in the skate park
because the expected value of the investment is a loss of $900.
14. Sample answer: The expected value of developing Product A is $115,000,
and the expected value of developing Product B is $88,000. The owner
should invest in Product A because it has a higher expected value of
earnings.
Practice and Problem
Solving Answers
6. Answers will vary. Sample
answer: Roll a number cube.
Let 1 or 2 represent Terrell, 3
or 4 represent Carmen, and 5
or 6 represent Alicia.
7. Answers will vary. Sample
answer: Assign a number
from 1 to 12 to each
salesperson and use a random
number generator to generate
2 integers from 1 to 12.
8. Answers will vary. Sample
answer: Assign a number
from 1 to 32 to each player
and use a random number
generator to generate 3
integers from 1 to 32.
9. Answers will vary. Sample
answer: Create a spinner with
4 equal spaces that represent
each special offer. Spin the
spinner until each special
offer has been spun at least
once, and stop as soon as
all 4 special offers have been
spun. Keep track of the results
in a frequency table. Conduct
several trials of the simulation
and find the average number
of purchases needed to get all
4 special offers.
10. Answers will vary. Sample
answer: Use a random
number generator to
generate integers from 1
through 100. Let 1 through
35 represent an ace and let
36 through 100 represent
a non-ace. Generate
numbers until there are
3 aces in a row and count
how many “serves” it took.
Conduct several trials of the
experiment and find the
average number of serves
needed to get 3 aces.
11.4 Probability Models
531
/HVVRQ 7KH1RUPDO'LVWULEXWLRQ
/
2EMHFWLYHV
2
8VHWKH(PSLULFDO5XOHWR
ILQGSUREDELOLWLHV
)LQG]VFRUHVDQGXVH
WKHVWDQGDUGQRUPDO
GLVWULEXWLRQ
7KHDPRXQWRISUHFLSLWDWLRQUHFHLYHGE\WKHFLW\RI&HGDU*URYH
HDFK\HDULVQRUPDOO\GLVWULEXWHGZLWKDPHDQRILQFKHVDQGD
VWDQGDUGGHYLDWLRQRILQFKHV,QWKLVOHVVRQ\RXZLOOOHDUQKRZWR
XVHSUREDELOLW\GLVWULEXWLRQVWRPDNHSUHGLFWLRQVDERXWIXWXUHHYHQWV
EDVHGRQSDVWUHVXOWV
7KH(PSLULFDO5XOH
$UDQGRPYDULDEOHLVDYDULDEOHZLWKDYDOXHWKDWLVGHWHUPLQHGE\D
UDQGRPHYHQW)RUH[DPSOHZKHQ\RXUROOGLFHWKHUDQGRPYDULDEOH
;FRXOGEHGH¿QHGDVWKHVXPRIWKHQXPEHUVUROOHG7KHQ;FDQWDNH
RQDQ\ZKROHQXPEHUEHWZHHQDQG$SUREDELOLW\GLVWULEXWLRQ
VKRZVWKHSUREDELOLW\RIHDFKSRVVLEOHRXWFRPHRIDUDQGRPHYHQW
Probability
1
—
6
1
—
9
1
—
18
0
2
3
4
5
6
7
8
9
10 11 12
Sum of Two Dice
7KHSUREDELOLW\GLVWULEXWLRQDERYHLVDGLVFUHWHSUREDELOLW\GLVWULEXWLRQ
EHFDXVHWKHUHDUHDGLVFUHWHQXPEHURISRVVLEOHRXWFRPHVIRUWKH
UDQGRPYDULDEOH2WKHUUDQGRPYDULDEOHVVXFKDVWKHZHLJKWVRIEDJV
RIFRQFUHWHDUHGHVFULEHGE\DFRQWLQXRXVSUREDELOLW\GLVWULEXWLRQ
EHFDXVHWKHYDULDEOHFDQWDNHRQDZLGHUDQJHRIYDOXHV7KHKHLJKWRI
HDFKEDULQWKHKLVWRJUDPUHSUHVHQWVDSUREDELOLW\VRWKHVXPRIWKH
DUHDVRIWKHEDUVLV
7KHGLVWULEXWLRQRIPDQ\UHDOZRUOGSKHQRPHQDFDQEHGHVFULEHG
E\WKHQRUPDOGLVWULEXWLRQ)RUH[DPSOHVKRHVL]HVRIKLJKVFKRRO
VWXGHQWVWKHQXPEHURIQDLOVLQDER[RIQDLOVRUWKHDPRXQWRIFRIIHH
GLVSHQVHGE\DFRIIHHPDFKLQHDUHDOOGLVWULEXWHGQRUPDOO\$QRUPDO
GLVWULEXWLRQLVDFRQWLQXRXVSUREDELOLW\GLVWULEXWLRQZLWKDEHOOVKDSH
WKDWLVFHQWHUHGDWDQGV\PPHWULFDERXWWKHPHDQ7KHVWDQGDUG
GHYLDWLRQLVDQXPHULFDOYDOXHWKDWGHVFULEHVKRZVSUHDGRXWDGDWD
VHWLVDERXWWKHPHDQ
&KDSWHU3UREDELOLW\DQG6WDWLVWLFV
532 Chapter 11 Probability and Statistics
7KH(PSLULFDO5XOH
$QRUPDOGLVWULEXWLRQZLWKPHDQ—DQGVWDQGDUG
GHYLDWLRQıKDVWKHIROORZLQJSURSHUWLHV
34%
34%
13.5%
13.5%
2.35%
2.35%
µ
µ±σ
µ ± 2σ
µ ± 3σ
‡ 7KHWRWDODUHDXQGHUWKHQRUPDOFXUYHLV
‡ $ERXWRIWKHGDWDOLHZLWKLQVWDQGDUG
GHYLDWLRQRIWKHPHDQ
‡ $ERXWRIWKHGDWDOLHZLWKLQVWDQGDUG
GHYLDWLRQVRIWKHPHDQ
‡ $ERXWRIWKHGDWDOLHZLWKLQVWDQGDUG
GHYLDWLRQVRIWKHPHDQ
([DPSOH 8VLQJWKH(PSLULFDO5XOH
6XSSRVHWKHKHLJKWVRIWKHIHPDOHVWXGHQWVDW3OHDVDQW5LGJH+LJK6FKRRO
DUHQRUPDOO\GLVWULEXWHGZLWKDPHDQRILQFKHVDQGDVWDQGDUGGHYLDWLRQ
RILQFKHV:KDWLVWKHSUREDELOLW\WKDWDUDQGRPO\VHOHFWHGIHPDOH
VWXGHQWIURPWKHVFKRROLVEHWZHHQDQGLQFKHV"
6ROXWLRQ
)LUVWGHWHUPLQHKRZPDQ\VWDQGDUGGHYLDWLRQVIURP
WKHPHDQKHLJKWVRIDQGLQFKHVUHSUHVHQW
±±± ȝ±ı
ȝı
8VHWKH(PSLULFDO5XOHWRDSSUR[LPDWH
WKHDUHDXQGHUWKHQRUPDOFXUYH
EHWZHHQȝ±ıDQGȝı
56.5
59
61.5
64
66.5
Height (in.)
69
71.5
7KH1RUPDO'LVWULEXWLRQ
11.5 The Normal Distribution
533
3”;” 6RWKHSUREDELOLW\WKDWDUDQGRPO\VHOHFWHGIHPDOHVWXGHQWLVEHWZHHQ
DQGLQFKHVWDOOLVDERXW
2QJRLQJ$VVHVVPHQW
:KDWLVWKHSUREDELOLW\WKDWDUDQGRPO\VHOHFWHGIHPDOHVWXGHQWIURPWKH
VFKRROLVEHWZHHQDQGLQFKHV"
7KH6WDQGDUG1RUPDO'LVWULEXWLRQ
7KHVWDQGDUGQRUPDOGLVWULEXWLRQLVWKHQRUPDOGLVWULEXWLRQZLWKPHDQ
DQGVWDQGDUGGHYLDWLRQ<RXFDQXVHWKHIRUPXODEHORZWRWUDQVIRUP
[YDOXHVIURPDQRUPDOGLVWULEXWLRQZLWKPHDQȝDQGVWDQGDUGGHYLDWLRQ
ıLQWR]YDOXHVIURPDVWDQGDUGQRUPDOGLVWULEXWLRQ7KH]YDOXHIRUD
SDUWLFXODU[YDOXHRIDUDQGRPYDULDEOHLVFDOOHGWKH]VFRUHIRUWKH[YDOXH
7KH]VFRUHUHSUHVHQWVWKHQXPEHURIVWDQGDUGGHYLDWLRQVWKH[YDOXHOLHV
DERYHRUEHORZWKHPHDQȝ
&DOFXODWLQJD]VFRUH
7RFRQYHUWDQ[YDOXHIURPDUDQGRPYDULDEOH
;WKDWLVQRUPDOO\GLVWULEXWHGZLWKPHDQ
ʅDQGVWDQGDUGGHYLDWLRQʍƚŽĂ]VFRUH
VXEWUDFWWKHPHDQIURPWKH[YDOXHDQG
GLYLGHWKHUHVXOWE\WKHVWDQGDUGGHYLDWLRQ
z=
x −µ
σ
([DPSOH )LQGLQJD]VFRUH
7KHSULFHVRIXVHGSLFNXSWUXFNVDWDORFDOGHDOHUVKLSDUHQRUPDOO\
GLVWULEXWHGZLWKDPHDQRIDQGDVWDQGDUGGHYLDWLRQRI:KDW
LVWKH]VFRUHIRUDXVHGSLFNXSWUXFNWKDWLVSULFHGDW"
6ROXWLRQ
8VHWKHIRUPXODWR¿QGWKHFRUUHVSRQGLQJ]VFRUH
x −µ
z=
)RUPXODIRUFRQYHUWLQJ]VFRUHV
σ
]=
17, 825 − 15, 800
1620
] 7KH]VFRUHLV
&KDSWHU3UREDELOLW\DQG6WDWLVWLFV
534 Chapter 11 Probability and Statistics
6XEVWLWXWH[ ȝ ı 6LPSOLI\
&ULWLFDO7KLQNLQJ:KDWGRHVWKH]VFRUHLQ([DPSOHWHOO\RXDERXWWKH
UHODWLRQVKLSEHWZHHQWKHVHOOLQJSULFHRIWKHWUXFNDQGWKHPHDQSULFH"7KH
VHOOLQJSULFHRIWKLVSDUWLFXODUWUXFNLVVWDQGDUGGHYLDWLRQVDERYHWKHPHDQSULFH
%\FRQYHUWLQJ[YDOXHVIURPDUDQGRPYDULDEOHWR]VFRUHVLWDOORZV\RX
WRHDVLO\FRPSDUHKRZWKHYDOXHVUHODWHWRWKHPHDQRIWKHSUREDELOLW\
GLVWULEXWLRQ7KLVDOVRDOORZV\RXWRXVHVWDQGDUGQRUPDOWDEOHVJUDSKLQJ
FDOFXODWRUVDQGVSUHDGVKHHWSURJUDPVWR¿QGWKHDUHDXQGHUWKHQRUPDO
FXUYHWKDWFRUUHVSRQGVWR]VFRUHVDQGVROYHSUREDELOLW\SUREOHPV
([DPSOH 8VLQJWKH6WDQGDUG1RUPDO'LVWULEXWLRQ
5HIHUWRWKHLQIRUPDWLRQJLYHQDWWKHEHJLQQLQJRIWKHOHVVRQ/HW;
UHSUHVHQWWKHDPRXQWRISUHFLSLWDWLRQ&HGDU*URYHUHFHLYHVHDFK\HDU:KDW
LVWKHSUREDELOLW\WKDWWKHFLW\ZLOOKDYHEHWZHHQDQGLQFKHVRI
SUHFLSLWDWLRQLQDIXWXUH\HDU"
6ROXWLRQ
)LQGWKHFRUUHVSRQGLQJ]VFRUHV
]1 =
48.68 − 42.6
37.64 − 42.6
= 1.9
= −1.55 ]2 =
3.2
3.2
8VHDJUDSKLQJFDOFXODWRUWR¿QGWKHDUHDXQGHUWKHVWDQGDUGQRUPDOFXUYH
EHWZHHQ] ±DQG] 7KHQRUPDOFGIFRPPDQGFDQEHXVHGWR
¿QGWKLVDUHD
PLACEHOLDER
6RWKHSUREDELOLW\WKDWDUDQGRPO\VHOHFWHGIXWXUH\HDULQ&HGDU*URYHZLOO
KDYHEHWZHHQDQGLQFKHVRISUHFLSLWDWLRQLVDERXW
7RHVWLPDWHWKHDUHDWRWKHOHIWRUULJKWRID]VFRUH\RXFDQXVH±DQG
DVWKHORZHUDQGXSSHUOLPLWUHVSHFWLYHO\7KLVLVEHFDXVHWKHUHLVVROLWWOH
DUHDXQGHUWKHVWDQGDUGQRUPDOFXUYHEH\RQGVWDQGDUGGHYLDWLRQVWKDWLW
LVQHJOLJLEOH)RUH[DPSOHQRUPDOFGI±±FRXOGEHXVHGWRHVWLPDWH
3=±
7KH1RUPDO'LVWULEXWLRQ
11.5 The Normal Distribution
535
Think and Discuss
Answers
1. Sample answer: The area
under the normal curve
represents the probability
that a random variable falls
in the range of values on the
x-axis under the curve.
2. Sample answer: If a z-score
is negative, the x-value is
less than the mean. If it is
positive, the x-value is greater
than the mean.
3. Subtract the mean from the
x-value and divide the result
by the standard deviation.
4. Sample answer: The z-score
tells you how many standard
deviations above or below
the mean the x-value lies. For
example a z-score of –2.2
tells you that the x-value
lies 2.2 standard deviations
below the mean.
5. Sample answer: You can
use standard normal tables,
calculators, and spreadsheet
programs to find probabilities
for the standard normal
curve.
Practice and Problem
Solving Answers
6.81.5%
7.95%
8.49.85%
9.15.85%
10.97.35%
11.–2.5
12.1.125
13.1.8
14.–3.1
536 Chapter 11 Probability and Statistics
2QJRLQJ$VVHVVPHQW
:KDWLVWKHSUREDELOLW\WKDWDUDQGRPO\VHOHFWHGIXWXUH\HDUZLOOKDYHOHVV
WKDQLQFKHVRISUHFLSLWDWLRQ"DERXW
/HVVRQ$VVHVVPHQW
7KLQNDQG'LVFXVVVHHPDUJLQ
:KDWLVWKHUHODWLRQVKLSEHWZHHQWKHDUHDXQGHUDQRUPDOFXUYH
DQGSUREDELOLW\"
:KDWGRHVWKHVLJQRID]VFRUHWHOO\RXDERXWDQ[YDOXHDQGWKH
PHDQ"
'HVFULEHKRZWRWUDQVIRUP[YDOXHVIURPDUDQGRPYDULDEOHZLWK
PHDQ—DQGVWDQGDUGGHYLDWLRQıLQWR]VFRUHV
:KDWGRHVWKH]VFRUHWHOO\RXDERXWUHODWLRQVKLSEHWZHHQDQ
[YDOXHDQGWKHPHDQRIDQRUPDOO\GLVWULEXWHGGDWDVHW"*LYHDQ
H[DPSOH
+RZGRHVWUDQVIRUPLQJ[YDOXHVLQWR]VFRUHVKHOS\RXFDOFXODWH
SUREDELOLWLHV"
3UDFWLFHDQG3UREOHP6ROYLQJVHHPDUJLQ
8VHWKH(PSLULFDO5XOHWRHVWLPDWHHDFKSUREDELOLW\
— SRXQGVı SRXQGV3OE”;”OE
— ı 3”;”
— PHWHUVı PHWHUV3P”;”P
— VDOHVı VDOHV3VDOHV”;”VDOHV
6XSSRVHWKHKHLJKWVRIWKHIHPDOHVWXGHQWVDW3OHDVDQW5LGJH
+LJK6FKRRODUHQRUPDOO\GLVWULEXWHGZLWKDPHDQRILQFKHV
DQGDVWDQGDUGGHYLDWLRQRILQFKHV:KDWLVWKHSUREDELOLW\
WKDWDUDQGRPO\VHOHFWHGIHPDOHVWXGHQWIURPWKHVFKRROLV
EHWZHHQDQGLQFKHV"
&RQYHUWHDFK[YDOXHWRD]VFRUH
[ — ı [ — ı [ — ı [ — ı &KDSWHU3UREDELOLW\DQG6WDWLVWLFV
8VHWKHVWDQGDUGQRUPDOGLVWULEXWLRQWRVROYHHDFKSUREOHP
7KHSULFHVRIODUJHVFUHHQ79VDWDQHOHFWURQLFVUHWDLOVWRUHDUH
QRUPDOO\GLVWULEXWHGZLWKDPHDQRIDQGDVWDQGDUG
GHYLDWLRQRI6XSSRVHDODUJHVFUHHQ79LVVHOHFWHGDW
UDQGRP:KDWLVWKHSUREDELOLW\WKDWWKHSULFHRIWKH79LV
EHWZHHQDQG"
7KHILQDOH[DPVFRUHVLQ0U+HU]RJ·VFODVVHVDUHQRUPDOO\
GLVWULEXWHGZLWKDPHDQRIDQGDVWDQGDUGGHYLDWLRQRI
,QRUGHUWRHDUQD%RQWKHH[DPDVWXGHQWPXVWKDYHDVFRUH
EHWZHHQDQG:KDWLVWKHSUREDELOLW\WKDWDUDQGRPO\
VHOHFWHGVWXGHQWHDUQHGD%RQWKHH[DP"
7KHDPRXQWRIWLPHLWWDNHVWKHILUHGHSDUWPHQWWRDUULYHRQ
VFHQHDIWHUEHLQJGLVSDWFKHGLVQRUPDOO\GLVWULEXWHGZLWKDPHDQ
RIPLQXWHVVHFRQGVDQGDVWDQGDUGGHYLDWLRQRIPLQXWH
VHFRQGV'XULQJWUDLQLQJH[HUFLVHVDILUHWUXFNLVGLVSDWFKHG
WRDUDQGRPO\VHOHFWHGDGGUHVVZLWKLQWKHGLVWULFW:KDWLVWKH
SUREDELOLW\WKDWWKHFUHZZLOODUULYHDWWKHDGGUHVVLQXQGHU
PLQXWHV"
7KHWDEOHVKRZVWKHPHDQZHLJKWDQGVWDQGDUGGHYLDWLRQIRUWZR
GLIIHUHQWEUDQGVRIFHUHDO
%UDQG$
%UDQG%
PHDQ
Practice and Problem
Solving Answers
15. about 85.4%
16. about 60.1%
17. about 65.5%
18. Brand B; The z-scores for
each box are 1.45 for Brand
A and –1.8 for Brand B. The
Brand A box is 1.45 standard
deviations above the mean
and the Brand B box is
1.8 standard deviations
below the mean.
VWDQGDUGGHYLDWLRQ
6XSSRVHDER[RI%UDQG$FHUHDOZHLJKVRXQFHVDQGD
ER[RIEUDQG%FHUHDOZHLJKVRXQFHV:KLFKER[ZHLJKWLV
IDUWKHUIURPWKHPHDQLQWHUPVRIVWDQGDUGGHYLDWLRQV"([SODLQ
KRZ\RXIRXQG\RXUDQVZHU
7KH1RUPDO'LVWULEXWLRQ
11.5 The Normal Distribution
537
MATH LAB
Activity 1
0DWK/DEV
PREPARE
• Remind students of the
meaning of a simulation.
Explain that this lab is a
simulation for how influenza
may spread throughout your
school or community.
• Discuss with students that in a
real situation of an infectious
disease, even though contact
might be made with an infected
person that is not a guarantee
of getting the infection.
TEACH
• This lab is a whole class activity.
• In a typical classroom setting,
the teacher will be activity
administrator and each
student will receive one
numbered note card.
• Keep the “infected” number
secret, as well as the identity
of the students that receives it.
• If the class is too small for this
simulation as it is described,
give each student two
numbered note cards. Instruct
the students to mentally assign
one of the numbers to the right
hand and the other number
to the left hand. When two
people come in immediate
contact, before rolling the
dice, have each person in the
pair choose right or left for the
other person. Each person then
identifies the ID number that
has been associated with the
right or left hand.
• The activity administrator
should be sure to change the
infected ID number and the
condition for being contagious
with each round. These
changes are necessary to get
the simulation as random as
possible.
$FWLYLW\6SUHDGRIDQ,QIHFWLRQ
3UREOHP6WDWHPHQW
(TXLSPHQW
,PDJLQHWKHUHLVDQRXWEUHDNRIWKHIOXDW
\RXUVFKRRO+RZOLNHO\LVLWWKDW\RXZLOO
EHLQIHFWHGE\WKHYLUXV"8VHDVLPXODWLRQ
WRPRGHOWKHVSUHDGRIWKHLQIHFWLRQ7KHQ
FDOFXODWHWKHH[SHULPHQWDOSUREDELOLW\RI
EHLQJLQIHFWHGE\WKHYLUXV
1RWHFDUGV
1XPEHUFXEHV
3URFHGXUH
7KHDFWLYLW\DGPLQLVWUDWRUGLVWULEXWHVDQRWHFDUGWRHDFK
VWXGHQWZLWKDQ,'QXPEHUVXFKDV«
2QHVWXGHQWZLOOUHFHLYHWKH,'QXPEHUWKDWKDVEHHQ
SUHVHOHFWHGWRUHSUHVHQWDVWXGHQWLQIHFWHGZLWKWKHIOXYLUXV
2QO\WKHDFWLYLW\DGPLQLVWUDWRUNQRZVZKLFKVWXGHQWKDVWKH
FDUGUHSUHVHQWLQJWKHIOXYLUXV
&DOFXODWHWKHWKHRUHWLFDOSUREDELOLW\WKDW\RXDUHWKHVWXGHQW
WKDWUHFHLYHVWKH,'QXPEHUFKRVHQWRUHSUHVHQWWKHLQIHFWHG
VWXGHQW
'XULQJHDFKURXQGRIWKHVLPXODWLRQPLQJOHZLWK\RXU
FODVVPDWHV<RXVKRXOGFRPHLQLPPHGLDWHFRQWDFWZLWK
ILYHVWXGHQWV3UHGLFWWKHQXPEHURIVWXGHQWVWKDWZLOOKDYH
FRQWUDFWHGWKHIOXYLUXVDIWHUWKUHHURXQGVRIWKHVLPXODWLRQ
3UHGLFWKRZPDQ\ZLOOEHLQIHFWHGDIWHUILYHURXQGVRIWKH
VLPXODWLRQ
(DFKVWXGHQWUHFHLYHVDQXPEHUFXEHWRXVHIRUSHUVRQDO
HQFRXQWHUV7KHDFWLYLW\DGPLQLVWUDWRUNQRZVWKHFRQGLWLRQIRU
SHUVRQDOHQFRXQWHUVZKHQWKHIOXLVFRQWDJLRXVEXWZLOOQRW
UHYHDOWKHFRQGLWLRQXQWLOODWHU$QH[DPSOHFRQGLWLRQPLJKW
EHZKHQWZRQXPEHUFXEHVDUHUROOHGLIWKHVXPLVRU
WKHQWKHFRQGLWLRQLVULJKWIRULQIHFWLQJVRPHRQHHOVHZLWKWKH
IOX,IWKHVXPLVDQRWKHUQXPEHUWKHQQRLQIHFWLRQFDQWDNH
SODFHIRUWKDWHQFRXQWHU
&KDSWHU3UREDELOLW\DQG6WDWLVWLFV
FOLLOW-UP
• Discuss the conditions used for contagious and list all possible rolls that
satisfied the condition. Determine if the condition chosen resulted in a
greater probability of being infected. Using all possible rolls, describe
a condition where very few people would likely become infected and a
condition where a lot of people would likely become infected.
Math Lab Solutions and Notes
2. The probability will vary depending on the number of students in the
simulation; 1 where n is the number of students in the simulation.
n
538 Chapter 11 Probability and Statistics
8VHDWDEOHVLPLODUWRWKHRQHVKRZQEHORZWRUHFRUGHDFK
SHUVRQDOHQFRXQWHU6WXGHQWVPRYHDURXQGWKHFODVVURRP
DQGPHHWZLWKRWKHUVWXGHQWV)RUHDFKHQFRXQWHUUHFRUG
WKHVWXGHQW·V,'QXPEHU7KHQHDFKVWXGHQWLQWKHSDLUUROOVD
QXPEHUFXEHDQGUHFRUGVWKHVXP&RQWLQXHPHHWLQJVWXGHQWV
XQWLOHDFKVWXGHQWKDVPHWZLWKILYHRWKHUVWXGHQWV%HVXUH
QRWWRHQFRXQWHUWKHVDPHVWXGHQWWZLFH
5RXQG
,'
6XP
$WWKHFRQFOXVLRQRI5RXQGWKHDFWLYLW\DGPLQLVWUDWRUUHYHDOV
WKH,'QXPEHURIWKHVWXGHQWRULJLQDOO\LQIHFWHGZLWKWKHIOX
YLUXV7KRVHVWXGHQWVZKRFDPHLQFRQWDFWZLWKWKLVVWXGHQW
VKRXOGFKHFNWKHLUVXPVIURPWKHHQFRXQWHUV,IWKHVXPRI
\RXUUROOZLWKWKH´LQIHFWHGµVWXGHQWZDVRUWKHQ
\RXZHUHJLYHQWKHIOX,I\RXUVXPZDVDQ\RWKHUQXPEHU
\RXGLGQRWFRQWUDFWWKHIOX&RXQWWKHQXPEHURILQIHFWHG
VWXGHQWVDWWKHFRQFOXVLRQRI5RXQG
'HWHUPLQHWKHWKHRUHWLFDOSUREDELOLW\RIFRQWUDFWLQJWKHIOX
RQFH\RXHQFRXQWHUWKH´LQIHFWHGVWXGHQWµ([SODLQ
)RU5RXQGWKHDFWLYLW\DGPLQLVWUDWRUUDQGRPO\DVVLJQVDQ
LQIHFWHG,'QXPEHUWRDQXPEHURIVWXGHQWVWKDWPDWFKHV
WKHFRXQWLQ6WHS)RUH[DPSOHLIWKHUHDUHWKUHHVWXGHQWV
LQIHFWHGDWWKHFRQFOXVLRQRI5RXQGWKHRULJLQDOVWXGHQW
SOXVWZRRWKHUVWKHQWKUHHVWXGHQWVVKRXOGEHVHOHFWHGDW
UDQGRPWREHLQIHFWHGDWWKHVWDUWRI5RXQG$JDLQWKH
DFWLYLW\DGPLQLVWUDWRULVWKHRQO\RQHZKRNQRZVWKHLQIHFWHG
,'QXPEHUV7KHDFWLYLW\DGPLQLVWUDWRUVKRXOGFKDQJHWKH
FRQGLWLRQIRUSHUVRQDOHQFRXQWHUVZKHQWKHIOXLVFRQWDJLRXV
EXWZLOOQRWUHYHDOWKHFRQGLWLRQXQWLOODWHU
&RQWLQXHUHSHDWLQJWKHDFWLYLW\IRUDWRWDORIWRURXQGVDV
WLPHSHUPLWV5HFRUGWKHQXPEHURILQIHFWHGVWXGHQWVDWWKH
HQGRIHDFKURXQG&RPSDUHWKHUHVXOWVRIWKHVLPXODWLRQZLWK
WKHSUHGLFWLRQWKDW\RXPDGHLQ6WHS
'HWHUPLQHWKHQXPEHURILQIHFWHGVWXGHQWVDWWKHFRQFOXVLRQ
RIWKHILQDOURXQG:KDWLVWKHH[SHULPHQWDOSUREDELOLW\RI
EHLQJLQIHFWHG"
0DWK/DEV
3 infected (from Round 1) encounter no previously infected. 3 × 5 = 15
possible newly-infected (7 or 8 expected to be newly-infected)
2 of 3 infected (from Round 1) encounter each other, and third infected
encounters no previously infected. 5 + 4 + 4 = 13 potential newly-infected (6
or 7 expected to be newly-infected)
1 of 3 infected (from Round 1) encounters both other infected people and
other 2 infected people do not encounter each other. 3 + 4 + 4 = 11
potential newly-infected people (5 or 6 expected to be newly-infected)
3 infected (from Round 1) each encounter other 2 infected. 3 + 3 + 3 = 9
potential newly-infected (4 or 5 expected to be newly-infected)
Math Lab Solutions
and Notes
6. In Round 1, each person
encounters 5, so infected
person has possibility of in­
fect­ing 5. Using theoretical
probability 1 , 5 • 1 = 2.5,
2
2
2 or 3 new people are
expected to be infected.
Expected number infected
after Round 1 is 1 + (2 or 3)
= 3 or 4.
1
7. ; 36 different outcomes
2
when two number cubes
rolled. Of those, 18 rolls have
sum of 7, 8, 9, or 10.
Table at bottom of following
page shows possible
dice combinations and
probabilities used. You
can alter the lab by using
different probabilities, such
as 1 or 1 . An example of
3
4
each is given; use the table to
find other conditions that will
result in desired probability.
1 = 12 condition with a
3
36
total of 12: 3, 4, 6, or 11
1 = 9 condition with a
4
36
total of 9: 2, 3, 4, or 10
8. During Round 2, if an
infected person encounters
another infected person,
the expected number of
newly-infected people will
be affected. Described at
the right are the 4 possible
scenarios based on 3 people
infected in Round 1. Note
that between 1 and 6 people
may be infected going into
Round 2, so the example
only considers one possible
result of Round 1 going into
Round 2.
Math Labs
539
MATH LAB
$FWLYLW\5RFN3DSHU6FLVVRUV
Activity 2
PREPARE
• Introduce the game Rock,
Paper, Scissors. Explain that
rock wins over scissors because
the rock can crush the scissors.
Scissors wins over paper
because scissors can cut paper.
Paper wins over rock because
paper can cover rock.
• Invite one student to come
to front with you and play a
coupled game. This will give
students a chance to better
understand the game before
beginning the lab.
5RFN
(TXLSPHQW
)ROORZWKHVWHSVEHORZWRFRPSDUHWKH
WKHRUHWLFDOSUREDELOLW\RIZLQQLQJDGRXEOH
HOLPLQDWLRQWRXUQDPHQWWRWKHH[SHULPHQWDO
SUREDELOLWLHV
&KDONERDUG
3DSHUDQG
SHQFLO
'LYLGHLQWRJURXSVRIIRXUVWXGHQWV'UDZDGRXEOHHOLPLQDWLRQ
WRXUQDPHQWEUDFNHWOLNHWKHRQHVKRZQEHORZRQDVKHHWRI
SDSHURURQWKHFKDONERDUG5DQGRPO\DVVLJQSRVLWLRQVIRU
HDFKSOD\HUIRUWKHILUVWWZRPDWFKHVLQWKH:LQQHU·V%UDFNHW
:ULWHWKHLQLWLDOVRIHDFKVWXGHQWRQWKHDSSURSULDWHOLQHRQ
WKHEUDFNHW
6FLVVRUV
FOLLOW-UP
540 Chapter 11 Probability and Statistics
3UREOHP6WDWHPHQW
3URFHGXUH
3DSHU
TEACH
• Students should work in
groups of 4 students.
• Each student will take on
the roles of participate and
recorder.
• Review the difference between
experimental probability and
theoretical probability.
• If the class size does not
accommodate groups of four,
ask students to consider a
difference format for playing
the tournament.
• Discuss how the activity
changes if it is a singleelimination tournament.
Talk about advantages and
disadvantages to playing
double elimination compare to
single elimination.
• Brainstorm different types
of tournaments and classify
each as more likely to be a
single elimination or double
elimination tournament.
5RFN3DSHU6FLVVRUVLVDJDPHSOD\HGEHWZHHQWZRSHRSOH,WLVRIWHQ
XVHGDVDGHFLVLRQPDNLQJJDPHVLPLODUWRIOLSSLQJDFRLQ2QWKH
FRXQWRIWKUHHHDFKSOD\HU´WKURZVµDURFNSDSHURUVFLVVRUVXVLQJ
KLVRUKHUKDQG$URFNLVIRUPHGE\PDNLQJDILVWSDSHUE\DIODW
SDOPDQGVFLVVRUVE\KROGLQJRXWWZRILQJHUV,QSOD\LQJWKHJDPH
5RFNEHDWV6FLVVRUV6FLVVRUVEHDWV3DSHUDQG3DSHUEHDWV5RFN,IWKH
SOD\HUVERWKWKURZWKHVDPHLWHPWKHJDPHLVUHSHDWHGXQWLOWKHUHLV
RQHZLQQHU
&KDSWHU3UREDELOLW\DQG6WDWLVWLFV
Sum of
dice
2
1+1
3
4
1 + 2; 1 + 3;
2 + 1; 3 + 1;
2 + 2;
5
6
7
8
9
10
11
12
1 + 4;
4 + 1;
3 + 2;
2 +3
1 + 5;
5 + 1;
2 + 4;
4 + 2;
3+3
1 + 6;
6 + 1;
2 + 5;
5 + 2;
3 + 4;
4+3
2 + 6;
6 + 2;
3 + 5;
5 +3;
4+4
4 + 5;
5 +4;
3 + 6;
6+3
4 + 6;
6 + 4;
5+5
5 + 6;
6+5
6+6
Number of
Rolls
1
2
3
4
5
6
5
4
3
2
1
Probability
1/36
1/18
1/12
1/9
5/36
1/6
5/36
1/9
1/12
1/18
1/36
,QRUGHUWREHHOLPLQDWHGIURPFRPSHWLWLRQLQDGRXEOH
HOLPLQDWLRQVW\OHWRXUQDPHQWDSOD\HUPXVWORVHWZRPDWFKHV
(YHU\SOD\HUEHJLQVLQWKH:LQQHU·V%UDFNHW7KHZLQQHUVRI
HDFKPDWFKLQWKHILUVWURXQGUHPDLQLQWKH:LQQHU·V%UDFNHW
ZKLOHWKHORVHUVRIWKHPDWFKHVDUHPRYHGWRWKH/RVHU·V
%UDFNHWZLWKRQHORVV,IDSOD\HUORVHVDPDWFKLQWKH/RVHU·V
%UDFNHWKHRUVKHKDVWZRORVVHVDQGLVHOLPLQDWHGIURPWKH
FRPSHWLWLRQ3OD\FRQWLQXHVXQWLODFKDPSLRQLVGHWHUPLQHG
3OD\VHYHUDOWRXUQDPHQWVZLWK\RXUJURXSRISOD\HUVDQG
UHFRUGWKHQXPEHURIWLPHV3OD\HURUZLQV$OVR
UHFRUGZKHWKHURUQRWWKHSOD\HUVLQWKHILQDOPDWFKKDG
DORVV%HVXUHWRUHGUDZIRUVWDUWLQJSRVLWLRQVDWWKHVWDUW
RIHDFKWRXUQDPHQW,IWLPHSHUPLWVWU\WRSOD\DWOHDVW
²WRXUQDPHQWV
&RPELQH\RXUUHVXOWVZLWKWKRVHRIWKHUHVWRIWKHFODVVLQD
WDEOHWRKDYHDODUJHSRRORIGDWD8VHWKLVGDWDWRILQGWKH
H[SHULPHQWDOSUREDELOLWLHVLQ6WHSVDQG
:KDWLVWKHWKHRUHWLFDOSUREDELOLW\RIZLQQLQJWKH5RFN3DSHU
6FLVVRUVWRXUQDPHQWZLWKRXWEHLQJVHQWWRWKH/RVHU·V%UDFNHW"
+RZGRHVWKHH[SHULPHQWDOSUREDELOLW\FRPSDUHZLWKWKH
WKHRUHWLFDOSUREDELOLW\"8VH3OD\HUDVDUHIHUHQFHLQHDFKWULDO
WRILQGWKHH[SHULPHQWDOSUREDELOLW\
:KDWLVWKHWKHRUHWLFDOSUREDELOLW\RIZLQQLQJWKH5RFN3DSHU
6FLVVRUVWRXUQDPHQWJLYHQWKDW\RXORVHRQHRI\RXUILUVWWZR
PDWFKHV"+RZGRHV\RXUH[SHULPHQWDOSUREDELOLW\FRPSDUH
ZLWKWKHWKHRUHWLFDOSUREDELOLW\"8VH3OD\HUDVDUHIHUHQFHLQ
HDFKWULDOWRILQGWKHH[SHULPHQWDOSUREDELOLW\
:KDWLVWKHWKHRUHWLFDOSUREDELOLW\RIEHLQJHOLPLQDWHGLQWZR
VWUDLJKWPDWFKHV"+RZGRHVWKHH[SHULPHQWDOSUREDELOLW\
FRPSDUHZLWKWKHWKHRUHWLFDOSUREDELOLW\"8VH3OD\HUDVD
UHIHUHQFHLQHDFKWULDOWRILQGWKHH[SHULPHQWDOSUREDELOLW\
0DWK/DEV
Math Labs
541
Math Applications
Solutions and Notes
0DWK$SSOLFDWLRQV
1. a.The options are inclusive
because he could have a
red, long-sleeved shirt.
b.There are 4 red longsleeved shirts and
23 possible shirts, so the
probability is 4 .
23
c.There are 11 long-sleeved
shirts and 5 other red
shirts, so the probability
is 16 .
23
d.There are 23 shirts,
16 pants, and 25 shoes,
so there are 23 • 16 • 25
= 9,200 different ways
Marcellus can dress his
avatar.
7KHDSSOLFDWLRQVWKDWIROORZDUHOLNHWKHRQHV\RXZLOOHQFRXQWHULQPDQ\ZRUNSODFHV8VHWKH
PDWKHPDWLFV\RXKDYHOHDUQHGLQWKLVFKDSWHUWRVROYHWKHSUREOHPV:KHUHYHUSRVVLEOHXVH
\RXUFDOFXODWRUWRVROYHWKHSUREOHPVWKDWUHTXLUHQXPHULFDODQVZHUV
*(1(5$/
&217(17
0DUFHOOXVMRLQHGDQRQOLQHJDPHLQZKLFKXVHUVFDQSDUWLFLSDWHLQDYLUWXDO
HFRQRP\E\H[FKDQJLQJGROODUVIRUDYLUWXDOFXUUHQF\SXUFKDVLQJYLUWXDOODQG
VHWWLQJXSYLUWXDOEXVLQHVVHVDQGEX\LQJDQGVHOOLQJYLUWXDOJRRGVDQGVHUYLFHV
ZLWKLQWKHHQYLURQPHQW7KHILUVWVWHSLQEHFRPLQJDPHPEHURIWKHRQOLQH
JDPHLVWRFKRRVHDQDYDWDUZKLFKLVDYLUWXDOUHSUHVHQWDWLRQRIWKHSOD\HU
:KHQFRQVWUXFWLQJKLVDYDWDU0DUFHOOXVLVILUVWDVNHGWRFKRRVHWKHVKLUWLW
ZLOOZHDU7KHW\SHVDQGFRORUVRIVKLUWVKHKDVWRFKRRVHIURPDUHOLVWHGLQWKH
WDEOHEHORZ
6KLUW7\SHVDQG&RORUV
VKRUWVOHHYHG
ORQJVOHHYHG
5HG
%OXH
%ODFN
0DUFHOOXVLVWKHQDVNHGWRFKRRVHDPRQJSRVVLEOHSDLUVRISDQWVDQG
SRVVLEOHSDLUVRIVKRHV6LQFH0DUFHOOXVFDQQRWGHFLGHKRZWRGUHVVKLV
DYDWDUKHGHFLGHVWRUDQGRPO\FKRRVHRQH+HWKHQGRHVWKHVDPHIRUWKH
SDLUVRISDQWVDQGSDLUVRIVKRHV
D 0DUFHOOXVFKRRVLQJDUHGVKLUWIRUKLVDYDWDUDQG0DUFHOOXVFKRRVLQJD
ORQJVOHHYHGVKLUWIRUKLVDYDWDUDUHWZRHYHQWV$UHWKHVHHYHQWVPXWXDOO\
H[FOXVLYHRULQFOXVLYH"LQFOXVLYH
E :KDWLVWKHSUREDELOLW\WKDWWKHVKLUW0DUFHOOXVFKRRVHVIRUKLVDYDWDULVD
UHGORQJVOHHYHGVKLUW":ULWHWKHDQVZHUDVDIUDFWLRQ 4
23
F :KDWLVWKHSUREDELOLW\WKDWWKHVKLUW0DUFHOOXVFKRRVHVIRUKLVDYDWDULVUHG
RUORQJVOHHYHG":ULWHWKHDQVZHUDVDIUDFWLRQ 16
23
G +RZPDQ\GLIIHUHQWZD\VFDQ0DUFHOOXVGUHVVKLVDYDWDULQRQHVKLUWRQH
SDLURISDQWVDQGRQHSDLURIVKRHV"
&KDSWHU3UREDELOLW\DQG6WDWLVWLFV
542 Chapter 11 Probability and Statistics
&DLWOLQZDQWVWRGHWHUPLQHKRZPXFKLQWHUHVWWKHUHLVLQVWDUWLQJDFKHVVFOXEDW
KHUVFKRRO+RZFDQVKHHIIHFWLYHO\JDWKHULQIRUPDWLRQRQWKLVVXEMHFW"$QVZHU
HDFKTXHVWLRQEHORZWRGHVLJQDVXLWDEOHVWDWLVWLFDOVWXG\VHHPDUJLQ
D :KDWLVWKHSRSXODWLRQDQGZKDWZLOOWKHVDPSOHRI&DLWOLQ·VVWXG\EH"
E :KDWTXHVWLRQLV&DLWOLQDWWHPSWLQJWRDQVZHU"
F :KDWW\SHRIVWDWLVWLFDOVWXG\VKRXOG&DLWOLQFRQGXFW"([SODLQ\RXU
UHDVRQLQJ
G 'HVFULEHWKHPHWKRGRORJ\RIWKHVWDWLVWLFDOVWXG\WKDW&DLWOLQFRXOGXVH
:RXOGWKHVWXG\OLNHO\JLYHELDVHGRUXQELDVHGUHVXOWV"([SODLQ
$ZHEVLWHKDVRYHUYLGHRVEHLQJXSORDGHGHYHU\GD\7KHZHEVLWH
FDWHJRUL]HVYLGHRVLQWRJURXSVVXFKDVFRPHG\PXVLFSHWVDQGDQLPDOV
DQGVFLHQFHDQGWHFKQRORJ\(DFKGD\WKHZHEVLWHIHDWXUHVWZRYLGHRVRQLWV
KRPHSDJHWKDWDUHUDQGRPO\FKRVHQIURPWKHYLGHRVWKDWKDYHEHHQXSORDGHG
WKDWGD\7KHYLGHRVDUHODEHOHG´)HDWXUHG9LGHRRIWKH'D\µDQG´2WKHU
1RWDEOH9LGHRµ2Q0RQGD\YLGHRVZHUHXSORDGHGWRWKHVLWHLQFOXGLQJ
FRPHG\YLGHRVPXVLFYLGHRVSHWVDQGDQLPDOVYLGHRVDQG
VFLHQFHDQGWHFKQRORJ\YLGHRV2Q0RQGD\WKHVLWHXSORDGHGYLGHRV
DOORIZKLFKDUHLQWKHVFLHQFHDQGWHFKQRORJ\JURXS
D :KDWLVWKHSUREDELOLW\WKDWQHLWKHUWKH´)HDWXUHG9LGHRRIWKH'D\µQRU
WKH´2WKHU1RWDEOH9LGHRµZLOOEHDSHWVDQGDQLPDOVYLGHR"5RXQGWRWKH
QHDUHVWWHQWKRIDSHUFHQW
E :KDWLVWKHSUREDELOLW\WKDWERWKWKH´)HDWXUHG9LGHRRIWKH'D\µDQGWKH
´2WKHU1RWDEOH9LGHRµZLOOEHHLWKHUDFRPHG\YLGHRRUDPXVLFYLGHR"
5RXQGWRWKHQHDUHVWWHQWKRIDSHUFHQW
F ,IWKHRQOLQHYLGHREURDGFDVWLQJZHEVLWHGHFLGHVWRFKRRVHWKH´)HDWXUHG
9LGHRRIWKH'D\µDQGWKH´2WKHU1RWDEOH9LGHRµDWUDQGRPIURPRQO\
WKHVFLHQFHDQGWHFKQRORJ\YLGHRVZKDWLVWKHSUREDELOLW\WKDWERWKWKH
´)HDWXUHG9LGHRRIWKH'D\µDQGWKH´2WKHU1RWDEOH9LGHRµZLOOEHRQHRI
8UVXOD·VYLGHRV"5RXQGWRWKHQHDUHVWWHQWKRIDSHUFHQW
0DWK$SSOLFDWLRQV
Math Applications
Solutions and Notes
2. a.Sample answer: The
population is all students in
her school, and the sample
is the students whose
opinions Caitlin will gather.
b.Sample answer: whether
or not there is enough
interest among the
students in her school to
start a chess club.
c.Sample answer: Caitlin
should conduct a survey
since she wants to gather
opinions.
d.Sample answer: Ask a
random selection of
5 students from each
homeroom in the school
the survey question,
“Would you be interested
in a school chess club?”
Analyze the results of the
survey to gauge student
interest. The survey would
produce unbiased results
because the sample is a
random sample and there
is no bias in the survey
question.
3,514  • 
3,513 
1−
3. a. 1−



 12,578   12,578 
≈ 0.519 ≈ 51.9%
b. 3, 278 + 3,113 • 3, 278 + 3,112
12,578
12,578
≈ 0.258 ≈ 25.8%
c. 274 ≈ 0.010 ≈ 1.0%
2, 674
Math Applications
543
Math Applications
Solutions and Notes
4. a. 725 = 29
3, 000 120
b. 725 − 1 = 724
3, 000 − 1 2, 999
c. 29 • 724 ≈ 0.058
120 2, 999
≈ 5.8%
D :KDWLVWKHSUREDELOLW\WKDWWKHILUVWZLQQLQJVWXGHQWFKRVHQLV
DVRSKRPRUH" 29
120
E $VVXPLQJWKDWWKHILUVWZLQQLQJVWXGHQWFKRVHQZDVDVRSKRPRUHZKDWLV
WKHSUREDELOLW\WKDWWKHVHFRQGZLQQLQJVWXGHQWLVDOVRDVRSKRPRUH" 724
2, 999
5. a.149 • 148 • 147
275 274 273
≈ 0.158 ≈ 15.8%
b.100% – 15.8% = 84.1%
c.0.158 • 0.158 • 0.842
≈ 0.021 ≈ 2.1%
d.0.158 • 0.158 • 0.158
≈ 0.004 ≈ 0.4%
&DQGDFHLVDPHPEHURIKHUXQLYHUVLW\·VEDVNHWEDOOWHDP+HUWHDPDGYDQFHG
WRWKH)LQDO)RXU%DVNHWEDOO7RXUQDPHQW7KHUHDUHQRWHQRXJKWLFNHWVDYDLODEOH
IRUDOORIWKHVWXGHQWVWRDWWHQGWKH)LQDO)RXUVRWKHXQLYHUVLW\GHFLGHGWRKROG
DORWWHU\WRFKRRVHZKLFKVWXGHQWVZLOOUHFHLYHWKHDYDLODEOHWLFNHWV6WXGHQWV
ZKRZHUHLQWHUHVWHGLQWLFNHWVZHUHLQVWUXFWHGWRILOORXWDIRUPZLWKWKHLU
QDPHDQGFODVVVWDQGLQJIUHVKPDQVRSKRPRUHMXQLRURUVHQLRU7KHUHZHUH
IUHVKPHQVRSKRPRUHVMXQLRUVDQGVHQLRUVLQWKHORWWHU\
F :KDWLVWKHSUREDELOLW\WKDWWKHILUVWZLQQLQJVWXGHQWFKRVHQDQGWKH
VHFRQGZLQQLQJVWXGHQWFKRVHQDUHERWKVRSKRPRUHV"5RXQGWRWKH
QHDUHVWWHQWKRIDSHUFHQW
)UDJUDQW)HUWLOL]HUVSHFLDOL]HVLQSOHDVDQWVPHOOLQJJURZWKHQKDQFLQJIHUWLOL]HU
IRUXVHRQDOOIUXLWDQGYHJHWDEOHV6L[W\GD\VDIWHUPDNLQJDVDOH)UDJUDQW
)HUWLOL]HUVHQGVRXWDFXVWRPHUVDWLVIDFWLRQVXUYH\UHTXHVWLQJIHHGEDFNRQLWV
SURGXFW,QRUGHUWRHQFRXUDJHFXVWRPHUVWRILOORXWDQGUHWXUQWKHVXUYH\
)UDJUDQW)HUWLOL]HUHQWHUVWKRVHFXVWRPHUVLQWRDUDQGRPGUDZLQJZKLFKDZDUGV
WKUHHSUL]HV)LUVWSUL]HLVDVHOISURSHOOHGIHUWLOL]HUVSUHDGHUDQGD\HDU·VVXSSO\
$*5,&8/785( RIIHUWLOL]HU6HFRQGSUL]HLVDSRVWHUDQG7VKLUW7KLUGSUL]HLVDNH\FKDLQ2IWKH
FXVWRPHUVZKRUHWXUQHGWKHVXUYH\ZHUHQHZFXVWRPHUVDQGZHUH
$*5,%86,1(66
UHSHDWFXVWRPHUV%HFDXVHWKHPDUNHWLQJGHSDUWPHQWZDQWVWRPDNHVXUHDW
OHDVWRQHQHZFXVWRPHUJHWVDSUL]HLWKDVGHFLGHGWRNHHSUHFRQGXFWLQJWKH
GUDZLQJXQWLODWOHDVWRQHQHZFXVWRPHULVFKRVHQDQGLI)UDJUDQW)HUWLOL]HUPXVW
UHFRQGXFWWKHGUDZLQJWKH\ZLOOSXWWKHFXVWRPHUVZKRZHUHSUHYLRXVO\GUDZQ
EDFNLQWRWKHSRRORISRWHQWLDOZLQQHUV
D :KDWLVWKHSUREDELOLW\WKDW)UDJUDQW)HUWLOL]HUFRQGXFWVWKHGUDZLQJPRUH
WKDQRQFH"5RXQGWRWKHQHDUHVWWHQWKRIDSHUFHQW
E :KDWLVWKHSUREDELOLW\WKDW)UDJUDQW)HUWLOL]HURQO\FRQGXFWVWKHGUDZLQJ
RQFH"5RXQGWRWKHQHDUHVWWHQWKRIDSHUFHQW
F )LQGWKHSUREDELOLW\WKDW)UDJUDQW)HUWLOL]HUFRQGXFWVWKHGUDZLQJWLPHV
5RXQGWRWKHQHDUHVWWHQWKRIDSHUFHQW
G )LQGWKHSUREDELOLW\WKDW)UDJUDQW)HUWLOL]HUFRQGXFWVWKHGUDZLQJPRUHWKDQ
WLPHV5RXQGWRWKHQHDUHVWWHQWKRIDSHUFHQW
&KDSWHU3UREDELOLW\DQG6WDWLVWLFV
544 Chapter 11 Probability and Statistics
%86,1(66
0$5.(7,1*
.HQGUDDQDWWRUQH\LVUHSUHVHQWLQJDQRQOLQHDGYHUWLVHUWKHSODLQWLIILQDFDVHRI
VXVSHFWHGFOLFNIUDXG&OLFNIUDXGLVDSUREOHPVSHFLILFWRWKHRQOLQHDGYHUWLVLQJ
LQGXVWU\·VSD\SHUFOLFNPRGHOLQZKLFKDGYHUWLVHUVGLUHFWO\RULQGLUHFWO\SD\WKH
SXEOLVKHUVRIWKHLURQOLQHDGVLHWKHZHEVLWHVRQZKLFKWKHLUDGVDSSHDUDQ
DJUHHGXSRQDPRXQWRIPRQH\HYHU\WLPHDXVHUFOLFNVRQRQHRIWKHLUDGV
6RPHWLPHVWKHVHFOLFNVDUHIUDXGXOHQW)RUH[DPSOHDSXEOLVKHUPD\LOOHJDOO\FOLFN
RQDQDGRQLWVZHEVLWHWRPDNHPRQH\IRULWVHOIDWWKHH[SHQVHRIWKHDGYHUWLVHU
7KHFRPSHWLWRUPD\LOOHJDOO\FOLFNRQWKHDGYHUWLVHU·VDGVWRGULYHXSLWVFRVWV
,QSUHSDUDWLRQIRUWKHWULDO.HQGUDKLUHGDQH[SHUWRQFOLFNIUDXGZKRKDV
GHWHUPLQHGWKDWRIWKHFOLFNVRQWKHSODLQWLII·VDGKDYHEHHQIUDXGXOHQWRU
LQRWKHUZRUGVWKHH[SHULPHQWDOSUREDELOLW\RIRQHRIWKHFOLFNVRQWKHSODLQWLII·V
DGEHLQJIUDXGXOHQWLV
D ,VHDFKFOLFNRQWKHSODLQWLII·VDGDQLQGHSHQGHQWHYHQWRUD
GHSHQGHQWHYHQW"LQGHSHQGHQWHYHQW
E ,IWKHH[SHUWRQFOLFNIUDXGZDVDQDO\]LQJVSHFLILFFOLFNVDQGWKHILUVW
FOLFNVZHUHIUDXGXOHQWZKDWZRXOGEHWKHSUREDELOLW\WKDWWKHIRXUWKFOLFN
ZRXOGEHIUDXGXOHQW"
F ,IWKHH[SHUWRQFOLFNIUDXGZDVDQDO\]LQJVSHFLILFFOLFNVDQGWKHILUVW
FOLFNVZHUHQRWIUDXGXOHQWZKDWZRXOGEHWKHSUREDELOLW\WKDWWKHIRXUWK
FOLFNZRXOGEHIUDXGXOHQW"
G :KDWLVWKHSUREDELOLW\RIFOLFNVLQDURZRQWKHSODLQWLII·VDGZRXOGEH
IUDXGXOHQW"5RXQGWRWKHQHDUHVWKXQGUHGWKSHUFHQW
0DUWLQDZRUNVIRUDFRPSDQ\WKDWPDNHVDQGVHOOVSHUVRQDOGLJLWDODVVLVWDQWV
3'$V0DUWLQDVDZWKHSURWRW\SHRIWKHFRPSDQ\·VODWHVWPRGHORI3'$ZKLFK
LVH[WUHPHO\VORZ0DUWLQDILUVWHVWDEOLVKHVDIRFXVJURXSPDGHXSRIW\SH$DQG
W\SH%SHUVRQDOLWLHV$SHUVRQZLWKDW\SH$SHUVRQDOLW\LVDOZD\VLQDUXVKDQG
RIWHQPXOWLWDVNVXQGHUWLPHFRQVWUDLQWVZKLOHDSHUVRQZLWKDW\SH%SHUVRQDOLW\
LVPRUHUHOD[HGDQGSDWLHQW0DUWLQD·VIRFXVJURXSFRQVLVWVRIW\SH$
SHUVRQDOLWLHVDQGW\SH%SHUVRQDOLWLHV0DUWLQDKDVWKHPHPEHUVRIWKHIRFXV
JURXSWU\RXWWKHSURWRW\SHRIWKHODWHVWPRGHORI3'$7KUHHRIWKHW\SH$
SHUVRQDOLWLHVJLYHLWDSRVLWLYHUHYLHZZKLOHRIWKHW\SH%SHUVRQDOLWLHVJLYHLW
DSRVLWLYHUHYLHZ
D :KDWLVWKHSUREDELOLW\WKDWDUDQGRPO\VHOHFWHGPHPEHURI0DUWLQD·VIRFXV
JURXSLVDW\SH%SHUVRQDOLW\RUWKHSHUVRQJDYHWKHSURWRW\SHRIWKHODWHVW
PRGHORI3'$DSRVLWLYHUHYLHZ" 25
Math Applications
Solutions and Notes
6. a.Because each click does
not depend on any
other event, they are
independent events.
b.Because each click
is independent, the
probability remains the
same: 0.14.
c.Because each click
is independent, the
probability remains the
same: 0.14.
d.0.14 • 0.14 • 0.14 • 0.14
≈ 0.0004 ≈ 0.04%
7. a.The events are inclusive,
so P(type B or positive)
= P(type B) + P(positive)
– P(type B and positive)
= 22 + 23 − 20 = 25
38 38 38 38
b.P(type A or negative) =
P(type A) + P(negative)
– P(type A and negative) =
16 + 15 − 13 = 18 = 9
38 38 38 38 19
38
E :KDWLVWKHSUREDELOLW\WKDWDUDQGRPO\VHOHFWHGPHPEHURI0DUWLQD·VIRFXV
JURXSLVDW\SH$SHUVRQDOLW\RUWKHSHUVRQGLGQRWJLYHWKHSURWRW\SHRI
WKHODWHVWPRGHORI3'$DSRVLWLYHUHYLHZ" 9
19
0DWK$SSOLFDWLRQV
Math Applications
545
Math Applications
Solutions and Notes
8. a.Sample answer: No, the
sample is representative of
the population since it is a
random sample.
$PDUNHWLQJFRPSDQ\FRQGXFWHGDVXUYH\RIDUDQGRPVDPSOHRIFRQVXPHUV
DQGDVNHGWKHPWRJLYHWKHLURSLQLRQVRQDQHZEUDQGRIVKDPSRR7KH
FRQVXPHUVZHUHDVNHGWRXVHWKHQHZEUDQGRIVKDPSRRIRUZHHNVDQG
WKHQWHOOZKHWKHUWKH\SUHIHUWKHQHZEUDQGSUHIHUWKHLUROGEUDQGRUKDYHQR
RSLQLRQ2IWKRVHFRQVXPHUVVXUYH\HGVDLGWKDWWKH\SUHIHUWKHQHZEUDQG
RIVKDPSRRVDLGWKDWWKH\VWLOOSUHIHUWKHLUROGEUDQGRIVKDPSRRDQG
FRQVXPHUVKDGQRRSLQLRQ
D 'R\RXWKLQNWKHUHLVDQ\ELDVLQWKHFRPSDQ\·VVWXG\"VHHPDUJLQ
b.1635
E +RZPDQ\FRQVXPHUVZHUHVXUYH\HGDOWRJHWKHU"
c.about 56.3%
F %DVHGRQWKHVXUYH\UHVXOWVZKDWLVWKHSUREDELOLW\WKDWDUDQGRPO\VHOHFWHG
FRQVXPHUZRXOGSUHIHUWKHQHZEUDQGRIVKDPSRR"DERXW
d.about 2.47%; between
53.8% and 58.8%
G 7KHPDUJLQRIVDPSOLQJHUURUGHVFULEHVKRZFORVHO\WKHSUHGLFWLRQVDERXW
WKHVWDWLVWLFVPDGHXVLQJDUDQGRPVDPSOHDUHWRWKHDFWXDOSDUDPHWHUV
RIWKHSRSXODWLRQ:KHQDUDQGRPVDPSOHRIQPHPEHUVLVWDNHQIURPD
SRSXODWLRQWKHPDUJLQRIVDPSOLQJHUURUFDQEHDSSUR[LPDWHGE\WKH
H[SUHVVLRQ r
:KDWLVWKHPDUJLQRIVDPSOLQJHUURULQWKHFRPSDQ\·V
Q
VXUYH\":KDWLVWKHLQWHUYDOWKDWFRQWDLQVWKHSRSXODWLRQSDUDPHWHURIWKH
SRUWLRQRIFRQVXPHUVZKRSUHIHUWKHQHZEUDQGRIVKDPSRR"VHHPDUJLQ
e.Sample answer: As the
sample size increases,
the margin of sampling
error decreases, and the
estimate of the population
parameter gets more
accurate. While increasing
the sample size will
produce more accurate
results, it may not be
feasible or cost effective to
sample a very large portion
of the population.
H :KDWKDSSHQVWRWKHPDUJLQRIVDPSOLQJHUURUDVWKHVDPSOHVL]H
LQFUHDVHV":KDWZRXOGVRPHRIWKHDGYDQWDJHVDQGGLVDGYDQWDJHVEHRI
LQFUHDVLQJWKHVDPSOHVL]HRIDVWXG\"VHHPDUJLQ
+($/7+
2&&83$7,216
-R\LVDYROXQWHHUIRUDZRUNRXWSURJUDP$VWXG\RI-R\·VZRUNRXWSURJUDP
IRXQGWKDWRIWKHSHRSOHFRPSOHWLQJWKHSURJUDPZLOOVWDUWZRUNLQJRXWRQ
DUHJXODUEDVLV,QRWKHUZRUGVWKHH[SHULPHQWDOSUREDELOLW\WKDWVRPHRQHZKR
FRPSOHWHVWKHSURJUDPZLOOVWDUWZRUNLQJRXWRQDUHJXODUEDVLVLV
D ,ISHRSOHZKRVXFFHVVIXOO\FRPSOHWHG-R\·VZRUNRXWSURJUDPZHUHFKRVHQ
DWUDQGRPZKDWLVWKHSUREDELOLW\WKDWDOORIWKHPZLOOVWDUWZRUNLQJRXW"
5RXQGWRWKHQHDUHVWWHQWKRIDSHUFHQW
E :KDWLVWKHSUREDELOLW\WKDWVRPHRQHZKRFRPSOHWHV-R\·VZRUNRXW
SURJUDPZLOOQRWVWDUWZRUNLQJRXW"
F ,ISHRSOHZKRVXFFHVVIXOO\FRPSOHWHG-R\·VZRUNRXWSURJUDPZHUHFKRVHQDW
UDQGRPZKDWLVWKHSUREDELOLW\WKDWWKHILUVWZLOOVWDUWZRUNLQJRXWDQGWKH
WKLUGZLOOQRWVWDUWZRUNLQJRXW"5RXQGWRWKHQHDUHVWWHQWKRIDSHUFHQW
G ,ISHRSOHZKRVXFFHVVIXOO\FRPSOHWHG-R\·VZRUNRXWSURJUDPZHUHFKRVHQ
DWUDQGRPZKDWLVWKHSUREDELOLW\WKDWWKHILUVWZLOOQRWVWDUWZRUNLQJRXW
WKHVHFRQGZLOOVWDUWZRUNLQJRXWDQGWKHWKLUGZLOOQRWVWDUWZRUNLQJRXW"
5RXQGWRWKHQHDUHVWWHQWKRIDSHUFHQW
&KDSWHU3UREDELOLW\DQG6WDWLVWLFV
546 Chapter 11 Probability and Statistics
)$0,/<
&21680(5
6&,(1&(
Math Applications
Solutions and Notes
&DQGLFHLVDSROLFHRIILFHU6KHVHWVXSDQGDGPLQLVWHUV
DVDIHW\FKHFNSRLQWWRPDNHVXUHGULYHUVZHDUWKHLU
VHDWEHOWV&DQGLFH·VSROLFHGHSDUWPHQWIRXQGWKDW
RIWKHPRWRULVWVWKDWWKH\VWRSDWDFKHFNSRLQWDUHQRW
ZHDULQJDVHDWEHOWKDYHQRYDOLGGULYHU·VOLFHQVH
DQGDUHQRWZHDULQJDVHDWEHOWDQGKDYHQRYDOLG
GULYHU·VOLFHQVH&DQGLFH·VSROLFHGHSDUWPHQWHVWLPDWHV
WKH\ZLOOVWRSPRWRULVWVDWWKHFKHFNSRLQW&DQGLFH
LVDGPLQLVWHULQJ
10. a.P(no seatbelt or no license)
= P(no seatbelt) + P(no
license) – P(no seatbelt
and no license) = 0.005 +
0.056 – 0.003 = 0.058
D :KDWLVWKHSUREDELOLW\WKDWDPRWRULVWVWRSSHGDWD
FKHFNSRLQWZLOOQRWEHZHDULQJDVHDWEHOWRUZLOOEH
GULYLQJZLWKRXWDYDOLGGULYHU·VOLFHQVH"
b.P(no seatbelt and no license)
= 0.003 • 2,564 ≈ 8
E +RZPDQ\PRWRULVWVVWRSSHGDWWKHFKHFNSRLQWFDQ&DQGLFH·VSROLFH
GHSDUWPHQWH[SHFWWRQRWEHZHDULQJDVHDWEHOWDQGEHGULYLQJZLWKRXWD
YDOLGGULYHU·VOLFHQVH"5RXQGWRWKHQHDUHVWPRWRULVW
11. c.Answers will vary. Sample
answer: The aggressive
growth plan has a greater
expected value, so Molly
might consider investing
in this plan if she still
has several years before
retirement. However,
if Molly is closer to
retirement age, she might
consider investing in the
moderate growth plan
since there is less of a risk
of losing money on her
investment.
0ROO\UHFHQWO\VWDUWHGDQHZMREDQGQHHGVWRUROORYHUKHUNUHWLUHPHQW
LQYHVWPHQWIURPKHUSUHYLRXVHPSOR\HU6KHLVFRQVLGHULQJLQYHVWLQJLQHLWKHU
D´PRGHUDWHJURZWKµRUDQ´DJJUHVVLYHJURZWKµPXWXDOIXQGSODQ7KHWDEOH
EHORZVKRZVWKHSUHGLFWHGRQH\HDUUHWXUQVRQDLQYHVWPHQWLQHDFKRI
WKHPXWXDOIXQGSODQV
2QH<HDU5HWXUQRQ,QYHVWPHQW
(FRQRPLF
&RQGLWLRQV
3UREDELOLW\
0RGHUDWH
*URZWK3ODQ
$JJUHVVLYH
*URZWK3ODQ
²
5HFHVVLRQ
²
6WDEOH(FRQRP\
6WURQJ(FRQRP\
(FRQRPLF%RRP
8VHWKHLQIRUPDWLRQLQWKHWDEOHWRDQVZHUHDFKTXHVWLRQ
D :KDWLVWKHH[SHFWHGYDOXHRIWKH\HDUUHWXUQRQDLQYHVWPHQW
ZLWKWKHPRGHUDWHJURZWKSODQ"
E :KDWLVWKHH[SHFWHGYDOXHRIWKH\HDUUHWXUQRQDLQYHVWPHQW
ZLWKWKHDJJUHVVLYHJURZWKSODQ"
F :KLFKJURZWKSODQZRXOG\RXUHFRPPHQG0ROO\XVHVZKHQVKHUROOVRYHU
KHUN":KDWRWKHUIDFWRUVPLJKW\RXQHHGWRNQRZEHIRUH\RXPDNHD
UHFRPPHQGDWLRQ"([SODLQVHHPDUJLQ
0DWK$SSOLFDWLRQV
Math Applications
547
Math Applications
Solutions and Notes
12. a.44 + 25 + 27 + 31
= 127; P = 127
235
:DOWHUUHFHQWO\UHDGDVWXG\RQFUHGLWFDUGLGHQWLW\WKHIW7KHVWXG\UHSRUWHGRQ
KRZORQJLWWRRNYLFWLPVWRUHVROYHWKHLUSUREOHPVEDVHGRQZKHWKHUWKHWKLHI
RSHQHGDQHZFUHGLWFDUGDFFRXQWLQWKHYLFWLP·VQDPHRUXVHGDQH[LVWLQJFUHGLW
FDUGDFFRXQW%HORZDUHWKHUHVXOWVRIWKHUDQGRPVWXG\RIFUHGLWFDUG
LGHQWLW\WKHIWYLFWLPV
1XPEHURI'D\VWR5HVROYH&UHGLW&DUG
,GHQWLW\7KHIW3UREOHP
1XPEHURI'D\V
²
²
!
1HZ&UHGLW&DUG
$FFRXQW
([LVWLQJ&UHGLW
&DUG$FFRXQW
D :KDWLVWKHSUREDELOLW\WKDWRQHRIWKHYLFWLPVLQWKHVWXG\KDGDQHZFUHGLW
FDUGDFFRXQWRSHQHGLQKLVKHUQDPHE\WKHWKLHI" 127
235
E :KDWLVWKHSUREDELOLW\WKDWRQHRIWKHYLFWLPVLQWKHVWXG\KDGDQHZFUHGLW
FDUGDFFRXQWRSHQHGLQKLVKHUQDPHE\WKHWKLHIDQGLWWRRNWKHYLFWLP
²GD\VWRUHVROYHWKHSUREOHP" 27
235
F :KDWLVWKHSUREDELOLW\WKDWLWWRRNWKHYLFWLPGD\VWRUHVROYHWKH
SUREOHPJLYHQDQHZFUHGLWFDUGDFFRXQWZDVRSHQHGLQWKHYLFWLP·VQDPH
E\WKHWKLHI" 27
127
7KHGULYHWKURXJKVHUYLFHWLPHVDWDIDVWIRRGUHVWDXUDQWDUHPHDVXUHGIURPWKH
WLPHDFXVWRPHUSXOOVXSWRWKHVHUYLFHZLQGRZWRWKHWLPHWKHFXVWRPHUSXOOV
DZD\7KHPDQDJHUDWDUHVWDXUDQWFODLPVWKDWWKHGULYHWKURXJKVHUYLFHWLPHV
DUHQRUPDOO\GLVWULEXWHGZLWKDPHDQRIVHFRQGVDQGDVWDQGDUGGHYLDWLRQRI
VHFRQGV8VHWKLVLQIRUPDWLRQWRDQVZHUHDFKTXHVWLRQ
D :KDWDUHWKHFRUUHVSRQGLQJ]VFRUHVIRUFXVWRPHUZDLWWLPHVRIVHFRQGV
DQGVHFRQGV"²DQG
E :KDWLVWKHSUREDELOLW\WKDWDUDQGRPO\VHOHFWHGGULYHWKURXJKFXVWRPHU
ZLOOZDLWEHWZHHQVHFRQGVDQGPLQXWH"
F 7KHPDQDJHU·VJRDOIRUWRGD\·VOXQFKUXVKLVDQDYHUDJHZDLWWLPHRI
VHFRQGVRIOHVV:KDWLVWKH]VFRUHIRUDZDLWWLPHRIVHFRQGV":KDWLV
WKHSUREDELOLW\WKDWDUDQGRPO\VHOHFWHGFXVWRPHUZLOOZDLWVHFRQGVRU
OHVV"
G ,IWKHPDQDJHUH[SHFWVWRVHUYHDERXWGULYHWKURXJKFXVWRPHUVGXULQJ
WRGD\·VOXQFKUXVKDERXWKRZPDQ\RIWKHPZRXOG\RXH[SHFWWRZDLW
VHFRQGVRUIHZHU"DERXWFXVWRPHUV
&KDSWHU3UREDELOLW\DQG6WDWLVWLFV
548 Chapter 11 Probability and Statistics
Math Applications
548
7KHWDEOHVKRZVWKHTXDOLW\UHVXOWVRIFDUGRRUDVVHPEOLHVGXULQJRQHVKLIWDWDQ
DXWRPRELOHDVVHPEO\SODQW
5HVXOW
,1'8675,$/
7(&+12/2*<
)UHTXHQF\
5HMHFWHG
1HHGV6OLJKW)L[
$SSURYHG
8VHWKHUHVXOWVLQWKHWDEOHWRDQVZHUHDFKTXHVWLRQ
D :KDWLVWKHH[SHULPHQWDOSUREDELOLW\WKDWWKHDVVHPEO\ZLOOEHUHMHFWHG
ZLOOQHHGDVOLJKWIL[RUZLOOEHDSSURYHG"5RXQGWRWKHQHDUHVWWHQWKRID
SHUFHQWZDONKLWRXW
E %DVHGRQWKHVHUHVXOWVLIWKHGRRUDVVHPEOLHVFDQEHIL[HGE\WKHHQGRIWKH
ZHHNKRZPDQ\DSSURYHGGRRUDVVHPEOLHVZRXOG\RXH[SHFWWKHUHWREHLI
WKHUHDUHVKLIWVGXULQJWKHZHHN"DERXW
F 6HWXSDQGFRQGXFWDVLPXODWLRQWRHVWLPDWHKRZPDQ\GRRUDVVHPEOLHV
ZLOOEHDSSURYHGLQWKHQH[WVKLIW'HVFULEHWKHSUREDELOLW\PRGHODQG
PHWKRGRORJ\XVHGVHHPDUJLQ
7KHVHUYLFHOLIHRIDFHUWDLQEUDQGRIOLJKWEXOELVQRUPDOO\GLVWULEXWHGZLWKD
PHDQRIKRXUVDQGDVWDQGDUGGHYLDWLRQRIKRXUV8VHWKLVLQIRUPDWLRQ
WRDQVZHUHDFKRIWKHIROORZLQJTXHVWLRQV
D 8VLQJWKH(PSLULFDO5XOHZKDWLQWHUYDOVUHSUHVHQWDQGRI
WKHVHUYLFHOLYHVRIWKLVEUDQGRIOLJKWEXOE"WRKRXUVWR
KRXUVWRKRXUV
Math Applications
Solutions and Notes
14. c.Answers will vary.
Sample answer: Use a
calculator and the function
randInt(1,89) 5 times to
generate 5 integers from
1 to 89. Let integers from
1 through 7 represent a
rejected assembly, from
8 through 34 represent
an assembly that needs
a slight fix, and from 35
through 89 represent
an approved assembly.
Conduct several trials of
the simulation and use
the results to predict how
many assemblies will be
approved in the next shift.
E 6XSSRVH&DURO\QLQVWDOOVRQHRIWKHOLJKWEXOEVLQKHUNLWFKHQOLJKW:KDW
LVWKHSUREDELOLW\WKDWWKHEXOEZLOOODVWEHWZHHQDQGKRXUV"
5RXQGWRWKHQHDUHVWWHQWKSHUFHQW
F $TXDOLW\FRQWUROHQJLQHHUVHOHFWVRIWKHOLJKWEXOEVDWUDQGRPIRU
WHVWLQJ+HLQVWDOOVWKHEXOEVLQVRFNHWVDQGPHDVXUHVWKHVHUYLFHOLYHVRI
WKHP2IWKRVHVDPSOHGDERXWKRZPDQ\ZRXOG\RXH[SHFWWRODVWORQJHU
WKDQKRXUV"DERXWOLJKWEXOEV
0DWK$SSOLFDWLRQV
Math Applications
549
Vocabulary Review
biased (11.3)
biased sample (11.3)
causation (11.3)
compound event (11.2)
conditional probability (11.2)
control group (11.3)
correlation (11.3)
dependent events (11.1)
expected value (11.4)
experiment (11.3)
fair decision (11.4)
Fundamental Theorem
of Counting (14.1)
inclusive events (11.2)
independent events (14.1)
mutually exclusive (11.2)
normal distribution (11.5)
observational group (11.3)
population (11.3)
probability distribution (11.5)
probability model (11.4)
randInt( (11.4)
random events (11.4)
random sample (11.3)
random variable (11.5)
sample (11.3)
simulation (11.4)
standard deviation (11.5)
standard normal distribution (11.5)
survey (11.3)
treatment group (11.3)
z-score (11.5)
&KDSWHU5HYLHZ
/HVVRQ
5HYLHZ([DPSOHV
$ER[FRQWDLQVUHGSHQVEOXHSHQVDQG
EODFNSHQV:KDWLVWKHSUREDELOLW\RI
SLFNLQJWZREOXHSHQVZLWKRXWUHSODFLQJWKH
¿UVWSHQ"
7DPP\KDVVNLUWVDQGEORXVHV+RZ
PDQ\GLIIHUHQWRXWILWVFDQVKHPDNHZLWK
VNLUWDQGEORXVH"
7KHUHDUHSHQVLQWKHER[DQGRIWKHPDUH
EOXH$VVXPLQJDEOXHSHQLVSLFNHG¿UVWWKHUH
$FDIHKDVHQWUpHVHOHFWLRQVGULQNV
ZLOOEHSHQVOHIWRIZKLFKDUHEOXH7KH
DQGVLGHLWHPV+RZPDQ\GLIIHUHQW
HYHQWVDUHGHSHQGHQW
OXQFKFKRLFHVDUHWKHUHIRUHQWUpH
7 6
GULQNDQGVLGHLWHP"
3EOXHWKHQEOXH 16 15
-RVLDKKDVUHGPDUEOHVEOXH
3EOXHWKHQEOXH 7
PDUEOHVJUHHQPDUEOHVDQG\HOORZ
40
PDUEOHV:KDWLVWKHSUREDELOLW\RI
7KHSUREDELOLW\RISLFNLQJWZREOXHSHQVZLWKRXW
7
SLFNLQJDUHGPDUEOHDEOXHPDUEOH
UHSODFLQJWKH¿UVWSHQLV 40
DJUHHQPDUEOHDQGD\HOORZPDUEOH
ZLWKRXWUHSODFHPHQW" 4
899
/HVVRQ
5HYLHZ([DPSOHV
<RODQGDWRVVHGDQXPEHUFXEHODEHOHG
WKURXJK:KDWLVWKHSUREDELOLW\WKDW
VKHUROOHGDQRGGQXPEHURUDQXPEHUOHVV
WKDQ"
6WXGHQW5HYLHZ([HUFLVHV
'LG\RXYRWHLQWKH
8VHWKHWDEOHDW
HOHFWLRQ"
WKHULJKWWRILQG
0DOH )HPDOH
HDFKSUREDELOLW\
7KHHYHQWV³RGG´DQG³OHVVWKDQ´FDQERWK
RFFXUDWWKHVDPHWLPHVRWKHHYHQWVDUH
LQFOXVLYHHYHQWV
3RGG 1
2
3OHVVWKDQ 1
3
3RGGDQGOHVVWKDQ 1 • 1 = 1
2 3 6
3$RU% 3$3%±3$DQG%
3RGGRUOHVVWKDQ 1 + 1 − 1 = 2
2 3 6 3
7KHSUREDELOLW\WKDW<RODQGDUROOHGDQRGG
QXPEHURUDQXPEHUOHVVWKDQLV 2 3
:KDWLVWKHSUREDELOLW\WKDWDSHUVRQ
YRWHGLQWKHHOHFWLRQJLYHQWKDWWKH
SHUVRQLVIHPDOH"
&KDSWHU3UREDELOLW\DQG6WDWLVWLFV
550 Chapter 11 Probability and Statistics
6WXGHQW5HYLHZ([HUFLVHV
)LQGWKHQXPEHURIRXWFRPHVLQ
HDFKVLWXDWLRQ
<HV
1R
:KDWLVWKHSUREDELOLW\WKDWDSHUVRQGLG
QRWYRWHLQWKHHOHFWLRQJLYHQWKDWWKH
SHUVRQLVPDOH"
1DQF\LVDIUHHWKURZVKRRWHU7KLV
PHDQVWKDWWKHSUREDELOLW\WKDWVKHZLOO
VXFFHVVIXOO\PDNHDIUHHWKURZLV
:KDWLVWKHSUREDELOLW\WKDW1DQF\ZLOO
VXFFHVVIXOO\PDNHKHUQH[WIUHHWKURZ
DWWHPSWVLQDURZ"5RXQGWRWKHQHDUHVW
WKRXVDQGWK
/HVVRQ²
5HYLHZ([DPSOHV
6WXGHQW5HYLHZ([HUFLVHV
;;;;;
;;;;;
;;;;;
; ;;;;;
&KDSWHU5HYLHZ
Chapter Review
551
&KDSWHU7HVW
)LQGWKHQXPEHURIRXWFRPHVLIRQHLWHP /XLJL·V3L]]HULDRIIHUVWKHVW\OHVDQG
LVVHOHFWHGIURPHDFKFDWHJRU\
WRSSLQJVVKRZQLQWKHWDEOH+RZPDQ\
GLIIHUHQWWRSSLQJSL]]DVDUHWKHUH"
VKLUWVUHGJUHHQEOXHEURZQZKLWH
SDQWVMHDQVVODFNV
/XLJL·V3L]]HULD
3L]]D6W\OH
+DQG7RVVHG
3DQ3L]]D
7KLQ&UXVW
6LFLOLDQ
VLGHGLVKEHDQVSRWDWRVDODGFROHVODZ
PHDWEXUJHUKRWGRJFKLFNHQ
GULQNVRIWGULQNWHDZDWHU
8VHWKHVXUYH\UHVXOWVLQWKHWDEOHWRILQG
HDFKSUREDELOLW\([SUHVVWKHDQVZHUV
DVGHFLPDOVURXQGHGWRWKHQHDUHVW
WKRXVDQGWK
5LJKW+DQGHG
/HIW+DQGHG
$PELGH[WURXV
0DOH
)HPDOH
3RQO\ULJKWKDQGHG_IHPDOH
3RQO\OHIWKDQGHG_PDOH
3DPELGH[WURXV
3RQO\OHIWKDQGHGRUDPELGH[WURXV
7RSSLQJV
3HSSHURQL
6DXVDJH
*UHHQ3HSSHUV
0XVKURRPV
%DFRQ
2QLRQV
*URXQG%HHI
7KHUHDUHJLUOVDQGER\VLQ0UV
'DGH·VJ\PFODVV6L[JLUOVSOD\DVSRUW
DQGJLUOVGRQRWSOD\DVSRUW1LQHER\V
SOD\DVSRUWDQGER\VGRQRWSOD\D
VSRUW,IDVWXGHQWLVVHOHFWHGDWUDQGRP
ZKDWLVWKHSUREDELOLW\WKDWWKHVWXGHQW
LVDJLUORUSOD\VDVSRUW"([SUHVVWKH
DQVZHUDVDIUDFWLRQ 23
30
;;;;;
(YDOXDWHHDFKH[SUHVVLRQ
;;;;;
;;;;;
;;;;;
;;;;;
;;;;;
7KHUHDUHUHGERZOLQJEDOOV
VSHFNOHGERZOLQJEDOOVDQGEODFN
ERZOLQJEDOOVRQWKHUDFNDWDERZOLQJ
DOOH\:KDWLVWKHSUREDELOLW\WKDWWKH
QH[WWZRFXVWRPHUVZLOOERWKVHOHFWUHG
ERZOLQJEDOOV"([SUHVVWKHDQVZHUDV
DIUDFWLRQ 38
255
&KDSWHU3UREDELOLW\DQG6WDWLVWLFV
552 Chapter 11 Probability and Statistics
$ER[FRQWDLQVQLQHJRRGOLJKWEXOEV
DQGIRXUGHIHFWLYHOLJKWEXOEV,I
0HDJDQVHOHFWVWKUHHOLJKWEXOEVDW
UDQGRPZKDWLVWKHSUREDELOLW\WKDW
WZRRIWKHPDUHJRRG"([SUHVVWKH
DQVZHUDVDIUDFWLRQ 24
133
7ZRFDUGVDUHGUDZQDWUDQGRPIURP
DGHFNRISOD\LQJFDUGV:KDWLVWKH
SUREDELOLW\WKDWERWKFDUGVDUHDFHV"
([SUHVVWKHDQVZHUDVDIUDFWLRQ 1
221
6WDQGDUGL]HG7HVW3UDFWLFH
0XOWLSOH&KRLFH
*LQDWRVVHVDQXPEHUFXEHODEHOHG
WKURXJK:KDWLVWKHSUREDELOLW\VKH
ZLOOUROODQHYHQQXPEHURUDQXPEHU
JUHDWHUWKDQ"([SUHVVWKHDQVZHUDVD
IUDFWLRQ
;;;;
2SHQ(QGHG5HVSRQVH
;;;;
$ ;;;;
& % ' 'DULQVHOHFWHGDQXPEHUDWUDQGRP
IURPWKHVHW^`
/HW$UHSUHVHQWWKHHYHQWVHOHFWLQJD
:KLFKRIWKHIROORZLQJWHUPVEHVW
PXOWLSOHRIDQG%UHSUHVHQWWKHHYHQW
GHVFULEHVWKHV\VWHPRIHTXDWLRQVEHORZ"
VHOHFWLQJDPXOWLSOHRI$UHWKHVHWZR
[\ ²&
HYHQWVPXWXDOO\H[FOXVLYH"([SODLQ)LQG
²[ \
3$DQG%DQG3$RU%VHHPDUJLQ
$ FRQVLVWHQW
% GHSHQGHQW
,VWKHVHWRISXUHO\LPDJLQDU\QXPEHUV
& LQFRQVLVWHQW
' LQGHSHQGHQW
FORVHGZLWKUHVSHFWWRPXOWLSOLFDWLRQ"
:KDWLVWKHYDOXHRIL"'
([SODLQDQGJLYHDQH[DPSOHWRVXSSRUW
$ L
% ²L
\RXUFODLPQRL‡L ²DQG²LVQRWD
SXUHO\LPDJLQDU\QXPEHU
& ' ²
(YHQWV$DQG%DUHLQGHSHQGHQW,I
3$ 1 DQG3% 1 ZKDWLV
4
2
3$DQG%"$
% 2
$ 1
8
5
' 3
& 2
3
4
;;;
$ ;;;;
& ;;;;
*ULGGHG5HVSRQVH
;;;;;;;;
;;;;
% ;;;;
' ;;;;
([WHQGHG5HVSRQVH
$VXUYH\RISHWRZQHUVUHYHDOHGWKDW
IHPDOHVRZQDGRJIHPDOHVRZQ
DFDWDQGIHPDOHVRZQERWK7KH
VXUYH\DOVRIRXQGWKDWPDOHVRZQD
GRJPDOHVRZQDFDWDQGPDOHV
RZQERWK
D &UHDWHDWDEOHWRRUJDQL]HWKHVXUYH\
UHVXOWVVHHPDUJLQ
Standardized Test
Practice Additional
Answers
Open Ended Response
9. The events are not mutually
exclusive because 60 is a
multiple of 3 and a multiple
of 4; P(A and B) = 1 ;
6
P(A or B) = 1
2
Extended Response
11. a.
What type of pet do you own?
Male
Female
Dog
42
35
Cat
30
24
Both
29
20
Standardized Test
Response Form (CRB)

E :KDWLVWKHSUREDELOLW\WKDWDSHW
RZQHUZKRSDUWLFLSDWHGLQWKHVXUYH\
RZQVRQO\DGRJJLYHQWKDWWKHSHW
RZQHULVIHPDOH"
F :KDWLVWKHSUREDELOLW\WKDWDSHW
RZQHUZKRSDUWLFLSDWHGLQWKHVXUYH\
RZQVERWKDGRJDQGDFDWJLYHQ
WKDWWKHSHWRZQHULVPDOH"
;;;;
&KDSWHU$VVHVVPHQWV
Chapter Assessments
553
Related documents